<<

December 2020 Current Affairs Best Revision Questions

Q) With reference to Shanghai Cooperation Organisation, consider the following statements: 1. Shanghai Cooperation Organisation was created in 2001 and and Pakistan became members in 2016. 2. The SCO's official languages are Russian and Chinese.

Which of the statements given above is/are correct?

A. 1 only B. 2 only C. Both 1 and 2 D. Neither 1 nor 2

ANS - B

Shanghai Cooperation Organisation was created in 2001 and India and Pakistan became members in 2017. SCO is a Eurasian political, economic and military organization aiming to maintain peace, security, and stability in the region. Cross-border terrorism is the biggest challenge for countries belonging to the Shanghai Cooperation Organisation said Vice-President M. Venkaiah Naidu while chairing a meeting of the SCO heads of government, comprising India, Russia, China, Pakistan, and four Central Asian states.

Q) Consider the following statements: 1. Amendments brought in the Central Motor Vehicles Rules were made for insertion of Feracryclum in the first aid kits. 2. Feracrylum is a hemostatic agent recently brought back by the Rajasthan Government.

Which of the statements given above is/are incorrect?

1 TELEGRAM LINK: https://t.me/opdemy WEBSITE: www.opdemy.com

A. 1 only B. 2 only C. Both 1 and 2 D. Neither 1 nor 2

ANS - C

The Rajasthan government’s Transport Department has brought back a bleeding-controlling ointment, Feracrylum, to the first aid kits of all motor vehicles being henceforth registered following a direction of the High Court, which had recently stayed a Central notification omitting the gel from the mandatory medical provisions in the vehicles.

Q) Consider the following statements: 1. South Africa was invited to join BRIC in December 2010, after which the group adopted the acronym BRICS. 2. The acronym "BRICS" was initially formulated in 2001 by economist Jim O'Neill, of Goldman Sachs

Which of the statements given above is/are correct?

A. 1 only B. 2 only C. Both 1 and 2 D. Neither 1 nor 2

ANS - C

BRICS is an acronym for the grouping of the world’s leading emerging economies, namely Brazil, Russia, India, China, and South Africa.

Q) Consider the following statements: 1. The ‘Cobra’ units are specially trained units of CISF to counter maoists. 2. Central Armed Police Forces of India works under the Ministry of Home Affairs for internal security.

Which of the statements given above is/are correct?

A. 1 only B. 2 only C. Both 1 and 2 D. Neither 1 nor 2

2 TELEGRAM LINK: https://t.me/opdemy WEBSITE: www.opdemy.com

ANS - B

The ‘Cobra’ units are specially trained units of CRPF to counter maoists. Originally constituted as the Crown Representative Police in 1939, it is one of the oldest Central paramilitary forces. After Independence, the force was renamed as Central Reserve Police Force by an Act of Parliament on December 28, 1949.

Q) Consider the following statements: 1. Lt Gen Harpal Singh has been appointed as the new Engineer in Chief of the Indian Army. 2. He is currently the Director-General of the Border Security Force.

Which of the statements given above is/are correct?

A. 1 only B. 2 only C. Both 1 and 2 D. Neither 1 nor 2

ANs - A

Lt Gen Harpal Singh is currently the Director-General of Border Road Organization.

Q) Consider the following statements: 1. Information Management and Analysis Centre (IMAC) is based in Mumbai. 2. It was formed in 2014.

Which of the statements given above is/are correct?

A. 1 only B. 2 only C. Both 1 and 2 D. Neither 1 nor 2

ANS - B

IMAC is based in Gurgaon, Uttar Pradesh. The Indian Navy’s Information Management and Analysis Centre (IMAC) will soon become a National Maritime Domain Awareness (NDMA) centre.

Q) Consider the following statements:

3 TELEGRAM LINK: https://t.me/opdemy WEBSITE: www.opdemy.com

1. Union Minister Nitin Gadkari has inaugurated and laid the foundation stones of 16 National Highway projects in Uttar Pradesh. 2. The construction cost of these infra projects is estimated to be around Rs 7477 crore.

Which of the statements given above is/are correct?

A. 1 only B. 2 only C. Both 1 and 2 D. Neither 1 nor 2

ANS - C

The highway projects include a road length of 505 kilometers and will provide better connectivity, convenience to the public as well as lead to economic growth in the state. National Highways works worth two lakh crore rupees are being undertaken in UP.

Q) Which word has been named "Word of the Year" in Cambridge Dictionary? A. Quarantine B. Lockdown C. Pandemic D. Test

ANS - A

Cambridge Dictionary has revealed its word of the pandemic-stricken year as “quarantine”. The dictionary's editors said that it was the third most searched word of the year, after the words “hello” and “dictionary”, which recur almost every year.

Q) US President Donald Trump recently exercised his powers under the Constitution to pardon Michael Flynn. What was his professional relationship with President Trump?

1. Defense Secretary 2. Chief Economic Advisor 3. National Security Advisor 4. None of the above

ANS - 3

With less than two months of his tenure remaining, US President Donald Trump exercised his powers under the Constitution to pardon Michael Flynn, his former National Security Advisor, who had twice

4 TELEGRAM LINK: https://t.me/opdemy WEBSITE: www.opdemy.com

pleaded guilty to lying to the FBI. The President of the US has the constitutional right to pardon or commute sentences related to federal crimes. The US Supreme Court has held that this power is “granted without limit” and cannot be restricted by Congress. Clemency is a broad executive power, and is discretionary — meaning the President is not answerable for his pardons, and does not have to provide a reason for issuing one.

Q) Which Indian satellite recently missed collision with a Russian earth observation satellite, Kanopus-V?

A. 2A B. Cartosat 3 C. Insat 3D D. Cartosat 2F

ANS - D

India's 700 kg cartography satellite Cartosat-2F and Russia's 450 kg Kanopus-V satellite had a near-miss in the outer space on Friday morning, said Roscosmos, Russia's state space corporation. Both the satellites designed for Earth's remote sensing were as close as 224 meters. ISRO chief K. Sivan later said that two satellites coming close to each other is a fairly regular occurrence, and such things aren't usually reported in public.

Q) Consider the following statements: 1. The Prime Minister Abiy Ahmed Ali of Eritrea was awarded the Nobel Peace prize 2019 for his efforts to resolve the 20-year-long border conflict between Ethiopia and Eritrea. 2. The Tigray Region is the northernmost region of Eritrea.

Which of the statements given above is/are incorrect?

A. 1 only B. 2 only C. Both 1 and 2 D. Neither 1 nor 2

ANS - C

Ethiopia’s Prime Minister Abiy Ahmed launched a large-scale military offensive against separatists in the restive Tigray province, which led to the deaths of possibly thousands and forced tens of thousands to flee as refugees, a familiar disquiet arose across the world.

5 TELEGRAM LINK: https://t.me/opdemy WEBSITE: www.opdemy.com

Q) With reference to Chardham Pariyojana, consider the following statements: 1. It is a programme taken up by the Ministry of Road Transport and Highways 2. It aims for connectivity improvement for Chardham - Kedarnath, Badrinath, Yamunotri and Gangotri in Uttarakhand.

Which of the statements given above is/are correct?

A. 1 only B. 2 only C. Both 1 and 2 D. Neither 1 nor 2

ANS - C

The cost of the project is around Rs. 12,000 Crore. The Implementing Agencies are Uttarakhand State Public Works Department (PWD), BRO and the National Highway & Infrastructure Development Corporation Limited (NHIDCL).

Q) Consider the following statements: 1. Serum Institute of India is the world’s 2nd largest vaccine maker, producing and selling more than 1.3 billion doses globally every year. 2. It was founded by Cyrus Poonawalla in 1966

Which of the statements given above is/are correct?

A. 1 only B. 2 only C. Both 1 and 2 D. Neither 1 nor 2

ANS - B

Serum Institute of India is the world’s largest vaccine maker, producing and selling more than 1.3 billion doses globally every year. The products developed include tuberculosis vaccine Tubervac (BCG), Poliovac for poliomyelitis, and other vaccinations for the childhood vaccination schedule.

Q) With reference to Purchasing Managers’ Index (PMI), consider the following statements: 1. PMI is an indicator of business activity in the manufacturing and services sectors. 2. The PMI is usually released at the start of every month and is considered a good leading indicator of economic activity.

Which of the statements given above is/are correct?

6 TELEGRAM LINK: https://t.me/opdemy WEBSITE: www.opdemy.com

A. 1 only B. 2 only C. Both 1 and 2 D. Neither 1 nor 2

ANS - C

The Purchasing Managers’ Index (PMI) for India’s manufacturing touched 56.3 in November, signaling that even as an improvement in wider industrial activity continued the sector’s expansion as well as the pace of of new orders slowed down while employment declined further as business optimism faded during the month.

Q) Consider the following statements: 1. DRDO delivered the biggest cryogenic propellant tank C32-LH2 to the Indian Space Research Organisation 2. Cryogenic fuels are fuels that require storage at extremely low temperatures in order to maintain them in a liquid state. Which of the statements given above is/are correct? A. 1 only B. 2 only C. Both 1 and 2 D. Neither 1 nor 2

ANS - B

The C32-LH2 tank is a developmental cryogenic propellant tank of aluminum alloy designed for improving the payload capability of GSLV MK-III launching vehicle of ISRO. The tank designed by HAL can load 5755 kg propellant fuel in the 89 cubic meter volume. Even earlier HAL had partnered with ISRO to supply satellite structures for the PSLV, GSLV Mark II, and III launch vehicles.

Q) Consider the following statements: 1. Annapurna idol was stolen from a temple of Tamil Nadu and smuggled out of the country around 100 years ago somewhere around 1913. 2. The 18th-century idol is part of the University of Regina, Canada’s collection at the MacKenzie Art Gallery.

Which of the statements given above is/are correct?

A. 1 only B. 2 only C. Both 1 and 2

7 TELEGRAM LINK: https://t.me/opdemy WEBSITE: www.opdemy.com

D. Neither 1 nor 2

ANS - B

In the November 29 episode of Mann Ki Baat, Prime Minister Narendra Modi announced that an ancient idol of the goddess Annapurna, stolen from India about a century ago, is being brought back from Canada. This idol was stolen from a temple of Varanasi and smuggled out of the country around 100 years ago somewhere around 1913. Annapurna, also spelled Annapoorna, is the goddess of food. The 18th-century idol, carved in the Benares style, is part of the University of Regina, Canada’s collection at the MacKenzie Art Gallery.

Q) Consider the following statements: 1. Lok Sabha Speaker Om Birla has appointed senior IAS officer Utpal Kumar Singh as Secretary-General of the Lok Sabha and Lok Sabha Secretariat. 2. He will succeed the incumbent Snehlata Shrivastava.

Which of the statements given above is/are correct?

A. 1 only B. 2 only C. Both 1 and 2 D. Neither 1 nor 2

ANS - C

Om Birla has appointed senior IAS officer Utpal Kumar Singh as Secretary-General of the Lok Sabha and Lok Sabha Secretariat. Retired IAS officer from the 1986 batch, Utpal Kumar Singh will replace Snehlata Shrivastava, the first woman Secretary-General of the Lok Sabha.

Q) Recently, an ancient idol of the Goddess Annapurna, stolen from India about a century ago, is being brought back from Canada. Maa Annapurna is the Goddess of what?

A. Goddess of Soil B. Goddess of Wind C. Goddess of Lightning D. Goddess of Food

ANS - D

8 TELEGRAM LINK: https://t.me/opdemy WEBSITE: www.opdemy.com

UPSC CSE PRELIMS 2021

CRASH COURSE DATE SUBJECT 10/04/2021, 11/04/2021 Budget and Economic Survey

12/04/2021, 13/04/2021 Art and Culture

14/0/2021, 15/04/2021, Economics 16/04/2021

17/18/2021,18/04/2021, Modern Indian History 19/04/2021

20/04/2021,21/04/2021, Geography 22/04/2021

23/04/2021,24/04/2021, Enviroment 25/04/2021

26/04/2021,27/04/2021, 28/04/2021,29/04/2021, Polity 30/04/2021

01/05/2021, 02/05/2021 Science and Tech *Note: Also, there will be Lectures on CSAT as and when time permits* PRICE: 1,999 For Any Queries: 7988797680, 7999136838, 6295118940

9 TELEGRAM LINK: https://t.me/opdemy WEBSITE: www.opdemy.com

Q) Union Home Minister Amit Shah visited the Bhagyalakshmi temple which is located in?

A. Kerala B. Tamil Nadu C. Telangana D. None of the above

ANS - C

Union Home Minister Amit Shah visited the Bhagyalakshmi temple while on a trip to Hyderabad. And Uttar Pradesh Chief Minister Yogi Adityanath made a pitch for renaming the city as Bhagyanagar. Bhagyalakshmi temple is a small temple dedicated to Goddess Lakshmi, adjacent to the southeast minar of Charminar. Made of bamboo poles and tarpaulins, it has a tin roof, and the southeast minar forms its back wall.

Q) Who has recently been appointed as the new Chairperson of National Dairy Development Board (NDDB)?

A. Varsha Joshi B. Mohammed Danish C. Utpal Kumar Singh D. Sunanda Sen

ANS - A

Q) With reference to the special session of Parliament, consider the following statements: 1. A special session is always dedicated to considering and passing a specific issue or bill. 2. To summon such as session, the government needs to first get both Houses Prorogued.

Which of the statements given above is/are correct?

A. 1 only B. 2 only C. Both 1 and 2

10 TELEGRAM LINK: https://t.me/opdemy WEBSITE: www.opdemy.com

D. Neither 1 nor 2

ANS - C

A special session is always dedicated to considering and passing a specific issue or bill. All other businesses like Question Hour, Zero Hour, etc, are dispensed with. To summon such as session, the government needs to first get both Houses prorogued.

Q) Consider the following statements: 1. India has invited the U.K. Prime Minister Boris Johnson as chief guest for the Republic Day celebrations in January 2021 2. President Cyril Ramaphosa was the chief guest in India for the Republic Day celebrations in 2020

Which of the statements given above is/are correct? A. 1 only B. 2 only C. Both 1 and 2 D. Neither 1 nor 2

ANS - A

India has invited the U.K. Prime Minister Boris Johnson as chief guest for the Republic Day celebrations in January 2021 and a decision on this is awaited from London, according to diplomatic sources. The invitation was extended last Friday during the telephone conversation between Prime Minister Narendra Modi and Mr. Johnson, a diplomatic source said on Wednesday. However, there was no confirmation from the British High Commission on this, with officials saying that Mr. Johnson was keen on visiting India as soon as possible.

Q) Consider the following statements: 1. The Chief Justice of India and the Judges of the Supreme Court are appointed by the President under clause (2) of Article 124 of the Constitution. 2. There has never been a woman Chief Justice of India.

Which of the statements given above is/are incorrect?

A. 1 only B. 2 only C. Both 1 and 2 D. Neither 1 nor 2

ANS - D

11 TELEGRAM LINK: https://t.me/opdemy WEBSITE: www.opdemy.com

Attorney-General (A-G) K.K. Venugopal on Wednesday took the Supreme Court’s invitation to suggest ways to gender-sensitise judges as an opportunity to tell the court it has never had a woman Chief Justice.

Q) With reference to National Investigation Agency (NIA), consider the following statements: 1. NIA is India's counter-terrorist task force. 2. National Investigation Agency (NIA) is Headquartered in Mumbai

Which of the statements given above is/are correct?

A. 1 only B. 2 only C. Both 1 and 2 D. Neither 1 nor 2

ANS - A

The National Investigation Agency (NIA) is India's counter-terrorist task force. The agency is empowered to deal with terror related crimes across states without special permission from the states. The Agency came into existence with the enactment of the National Investigation Agency Act 2008 by the Parliament of India on 31 December 2008, which was passed after the deadly 26/11 terror attack in Mumbai.

Q) Approval to the Pfizer-BioNTech vaccine, which they say is 95% effective in preventing illness has recently been done by which country ?

A. USA B. Singapore C. Japan D. Britain

ANS - D

Britain approved Pfizer’s COVID-19 vaccine on Wednesday, jumping ahead of the rest of the world in the race to begin the most crucial mass inoculation programme in history with a shot tested in wide-scale clinical trials.

Q) Consider the following statements: 1. Footwear major Bata Shoe Organization has appointed Sandeep Kataria as the new global Chief Executive Officer (CEO).

12 TELEGRAM LINK: https://t.me/opdemy WEBSITE: www.opdemy.com

2. He is the first Indian to be appointed to that post by Bata.

Which of the statements given above is/are correct?

A. 1 only B. 2 only C. Both 1 and 2 D. Neither 1 nor 2

ANS - C

Q) Which State government has launched a campaign titled ‘Duare Sarkar’ (government at doorsteps) to aware people about the 11 state-run welfare schemes and help them avail its benefits? A. West Bengal B. Assam C. Kerala D. Orissa

ANS - A

Q) A "child-friendly" police station set up in accordance with the guidelines laid down by the National Commissioner for Protection of Child Rights (NCPCR) was recently launched in which city?

A. Ahmedabad B. C. Mumbai D.

ANS - D

Pune police have collaborated with Hope for Children Foundation to set up the police station. The child-friendly police station is an innovative concept in the direction of preventing or tackling juvenile crimes and children reforms.

Q) With reference to the right to privacy, consider the following statements: 1. Indians have a constitutionally protected fundamental right to privacy that is an intrinsic part of life and liberty under Article 15 2. Right to privacy was held under Justice K. S. Puttaswamy (Retd) Vs Union of India in 2017

Which of the statements given above is/are correct?

13 TELEGRAM LINK: https://t.me/opdemy WEBSITE: www.opdemy.com

A. 1 only B. 2 only C. Both 1 and 2 D. Neither 1 nor 2

ANS - B

Indians have a constitutionally protected fundamental right to privacy that is an intrinsic part of life and liberty under Article 21

Q) Consider the following statements: 1. Lottery, gambling, and betting are taxable under the Goods and Services Tax (GST) Act. 2. Lottery, betting and gambling were “actionable claims” and came within the definition of ‘goods’ under Section 2(52) of the Central Goods and Services Tax Act, 2017. Which of the statements given above is/are correct?

A. 1 only B. 2 only C. Both 1 and 2 D. Neither 1 nor 2

ANS - C

The Supreme Court on Thursday held that lottery, gambling and betting is taxable under the Goods and Services Tax (GST) Act. The court held that lottery, betting and gambling were “actionable claims” and came within the definition of ‘goods’ under Section 2(52) of the Central Goods and Services Tax Act, 2017. A three-judge Bench led by Justice Ashok Bhushan said the levy of GST on lotteries did not amount to “hostile discrimination”.

Q) Which municipal corporation listed at the Bombay Stock Exchange became the first in north India to have raised municipal bonds?

A. Ghaziabad Municipal Corporation B. Allahabad Municipal Corporation C. Lucknow Municipal Corporation D. Ayodhya Municipal Corporation

ANS - C Lucknow Municipal Corporation bonds on December 2 became the first municipal bonds from North India to be listed on Bombay Stock. Exchange (BSE). Uttar Pradesh Chief Minister Yogi Adityanath attended the listing ceremony in Mumbai.

14 TELEGRAM LINK: https://t.me/opdemy WEBSITE: www.opdemy.com

Q) With reference to the Joint Comprehensive Plan of Action (JCPOA), consider the following statements: 1. JCPOA seeks to prevent Iran from producing a nuclear weapon. 2. JCPOA is an agreement signed by Iran and the P5+1+1 (China, France, Germany, Russia, the United Kingdom, the United States, and EU) on July 14, 2015.

Which of the statements given above is/are correct?

A. 1 only B. 2 only C. Both 1 and 2 D. Neither 1 nor 2

ANS - C

The Joint Comprehensive Plan of Action (JCPOA) known commonly as the Iran nuclear deal or Iran deal is an agreement on the Iranian nuclear the program reached in Vienna on 14 July 2015, between Iran and the P5+1 (the five permanent members of the United Nations Security Council—China, France, Russia, United Kingdom, United States—plus Germany) together with the European Union.

Q) With reference to Global Terrorism Index 2020, consider the following statements: 1. Afghanistan has topped the index as the worst terror impacted nation among the 163 countries. 2. India has been ranked 5th globally in the list of countries most affected by terrorism.

Which of the statements given above is/are correct?

A. 1 only B. 2 only C. Both 1 and 2 D. Neither 1 nor 2

ANS - A

The Global Terrorism Index is published annually by the Institute for Economics and Peace (IEP). Headquartered in Sydney, Australia, the institute measures the impact of terrorism on many countries. India has been ranked 8th globally in the list of countries most affected by terrorism.

Q) Consider the following statements: 1. A 11.5-megawatt power plant based on municipal waste has been set up in Bidadi, Bengaluru will convert 600 tonnes of municipal waste into 11.5-megawatt

15 TELEGRAM LINK: https://t.me/opdemy WEBSITE: www.opdemy.com

energy. 2. The power plant project is being constructed at a cost of Rs. 260 crore and will be operational by 2022.

Which of the statements given above is/are correct?

A. 1 only B. 2 only C. Both 1 and 2 D. Neither 1 nor 2

ANS - C

It is the first of its kind power plant in the state being set up by the Karnataka Power Corporation Limited (KPCL). This power plant will add 80.59 million units of power to the existing power grid. Chief Minister of Karnataka: B.S. Yediyurappa Governor: Vajubhai Vala

Q) Who has been appointed as the 27th Director General of Border Roads Organisation (BRO)?

A. Lt Gen Harpal Singh B. Lt Gen Rajeev Chaudhary C. Lt Gen Sodhi Singh Bassi D. Lt Gen Paramjit Singh Sangha

ANS - B

Lt Gen Chaudhary has succeeded Lt Gen Harpal Singh who has been appointed as the new Engineer-in-Chief of the Indian Army. The BRO is responsible for construction and maintenance of all border roads including those along the frontiers with China and Pakistan.

Q) With which bank has the Government of India inked $50 million policy-based loan agreement to improve financial management procedures and operational efficiencies in West Bengal?

A. New Development Bank B. World Bank C. Asian Development Bank D. Organisation for Economic Co-operation and Development

ANS - C

The Asian Development Bank is a regional development bank established on 19

16 TELEGRAM LINK: https://t.me/opdemy WEBSITE: www.opdemy.com

December 1966. The company also maintains 31 field offices around the world for social and economic development in Asia.

Headquarters: Mandaluyong, Philippines President: Masatsugu Asakawa Founded: 19 December 1966

Q) Consider the following statements: 1. Union Minister for Petroleum and Natural Gas and Steel, Dharmendra Pradhan launched India’s first indigenously developed ‘100 Octane premium petrol’ 2. Octane ratings are the measures of how much volatile fuel is.

Which of the statements given above is/are correct?

A. 1 only B. 2 only C. Both 1 and 2 D. Neither 1 nor 2

ANS - A

Octane ratings are measures of fuel stability that increases engine efficiency. Note - The high octane premium grade petrol will be marketed under the brand known as ‘XP-100’. It is developed by Indian Oil Corporation Limited (IOCL) at its Mathura refinery in Uttar Pradesh. India became the 7th country in the world to have such quality of fuels, making it into an elite club of countries. The other 6 countries that have access to this fuel are the USA, Germany, Greece, Indonesia, Malaysia, and Israel.

Q) The Reserve Bank of India (RBI) has recently ordered which bank to halt its digital launches with regard to certain incidents of outages in the internet banking, mobile banking and payment utilities of the lender over the past 2 years?

A. HDFC Bank B. Punjab National Bank C. Yes Bank D. PMC Bank

ANS - A

Q) With reference to District Development Councils (DDC), consider the following statements: 1.The amendment in Jammu and Kashmir Panchayati Raj Act, 1989 facilitate the

17 TELEGRAM LINK: https://t.me/opdemy WEBSITE: www.opdemy.com

setting up of District Development Councils (DDC). 2. The term of the DDC will be three years

Which of the statements given above is/are correct?

A. 1 only B. 2 only C. Both 1 and 2 D. Neither 1 nor 2

ANS - A

The term of the DDC will be five years. This system effectively replaces the District Planning and Development Boards in all districts, and will prepare and approve district plans and capital expenditure.

Q) Consider the following statements: 1. Lakshadweep is the smallest Union Territory of India 2. Kavaratti serves as the capital of the Union Territory and the region comes under the jurisdiction of Karnataka High Court.

Which of the statements given above is/are correct?

A. 1 only B. 2 only C. Both 1 and 2 D. Neither 1 nor 2

ANS - A

Kavaratti serves as the capital of the Union Territory and the region comes under the jurisdiction of the Kerala High Court. The name Lakshadweep was adopted on 1 November 1973 These islands are separated from one another by very narrow straits. The Lakshadweep Islands group is a Union Territory administered by the President through a Lt. Governor.

Q) Name the cyclone to lash Sri Lanka’s Northern and Eastern provinces ?

A. Nivar B. Burevi C. Amphan D. Nisarga

18 TELEGRAM LINK: https://t.me/opdemy WEBSITE: www.opdemy.com

ANS - B

At least 12,000 residents of Sri Lanka’s Northern and Eastern provinces were severely affected as heavy rain lashed the region while cyclone Burevi made a landfall on Wednesday night.

Q) Consider the following statements: 1. Shantiniketan in West Bengal, was established by Maharshi Devendranath Tagore. 2. Shantiniketan is on the UNESCO's Tentative List since 2010

Which of the statements given above is/are correct?

A. 1 only B. 2 only C. Both 1 and 2 D. Neither 1 nor 2

ANS - C

Shantiniketan was established by Maharshi Devendranath Tagore, and later expanded by his son Rabindranath Tagore whose vision became what is now a university town with the creation of Visva-Bharati?

Q) Consider the following statements: 1. The 9th edition of the International Sand Art Festival and the 31st edition of the Konark Festival has kicked off in Odisha. 2. International Sand Art Festival is being held at the Chandrabhaga Beach of Konark in Puri district of Odisha.

Which of the statements given above is/are correct?

A. 1 only B. 2 only C. Both 1 and 2 D. Neither 1 nor 2

ANS - C

About 70 artists from across the country are taking part in the festival. The world-renowned sand artist and Padmashree Awardee, Sudarsan Pattnaik has been appointed as Chief Curator of the festival.

Q) Consider the following statements: 1. The Maratha Emperor, Chhatrapati Shivaji Bhosle of the 17th century is

19 TELEGRAM LINK: https://t.me/opdemy WEBSITE: www.opdemy.com

considered as “Father of the Indian Navy”. 2. The theme of Navy Day 2020 is “Indian Navy Combat Ready, Credible & Cohesive”.

Which of the statements given above is/are incorrect?

A. 1 only B. 2 only C. Both 1 and 2 D. Neither 1 nor 2

ANS - D

The National Navy Day is observed every year on 4th December, to celebrate the achievements and role of the naval force of the country. Chief of Naval Staff - Admiral Karambir Singh Vice Chief of Naval Staff - Vice Admiral G Ashok Kumar Indian Navy Founded: 26th January 1950

Q) The Ministry of Home Affairs (MHA) released a list of the top 10 police stations in India for the year 2020. Which police station has topped the list?

A. Kharsang (Changlang, Arunachal Pradesh) B. Jhilmili (Surajpur, Chhattisgarh) C. Nongpok Sekmai (Thoubal, Manipur) D. AWPS-Suramangalam (Salem, Tamil Nadu)

ANS - C

Q) Which company has topped the 2020 Fortune 500 list of Indian companies?

A. Reliance Industries Limited B. Indian Oil Corporation Ltd C. Oil and Natural Gas Corporation D. Tata Motors

ANS - A

The country’s biggest oil firm, Indian Oil Corporation Ltd (IOC), was given the second spot, followed by Oil and Natural Gas Corporation (ONGC) at the third position. The list was published by Fortune India, which is part of the Kolkata-based RP Sanjiv Goenka Group.

Q) Consider the following statements: 1. Kuldeep Handoo has been appointed as the Ambassador of Fit India movement. 2. He is a cricket coach based in Srinagar, Jammu & Kashmir.

20 TELEGRAM LINK: https://t.me/opdemy WEBSITE: www.opdemy.com

Which of the statements given above is/are incorrect?

A. 1 only B. 2 only C. Both 1 and 2 D. Neither 1 nor 2

ANS - B

He is a Wushu Coach, a hard and soft and complete martial art, as well as a full-contact sport. It has a long history in reference to Chinese martial arts.

Q) With which country has the Indian Government recently signed a Memorandum of Understanding (MoU) in the field of Intellectual Property Cooperation?

A. Japan B. USA C. Maldives D.

ANS - B

The MoU was signed between Dr. Guruprasad Mohapatra, Secretary, DPIIT, and Mr. Andrei Iancu, who represented the Department for Promotion of Industry and Internal Trade (DPIIT), Ministry of Commerce and Industry and the United States Patent and Trademark Office (USPTO), Department of Commerce of the United States of America respectively.

Q) With reference to Elephant corridors, consider the following statements: 1.Elephant corridors allow elephants to continue their nomadic mode of survival, despite shrinking forest cover, by facilitating travel between distinct forest habitats. 2. ‘Gaj Yatra’ is a nationwide campaign to protect elephants launched on the occasion of World Elephant Day in 2017.

Which of the statements given above is/are correct?

A. 1 only B. 2 only C. Both 1 and 2 D. Neither 1 nor 2

21 TELEGRAM LINK: https://t.me/opdemy WEBSITE: www.opdemy.com

ANS - C

The National Green Tribunal (NGT) has directed the Odisha government to prepare an action plan within three months on 14 identified elephant corridors for providing stress-free migration to jumbos from one habitation to another in the State.

Q) With reference to Hampi chariot, consider the following statements: 1. Hampi reflects skills of temple architecture under the patronage of Vijayanagara rulers. 2. The Rs 20 denomination notes have the motif of Hampi with Chariot on the reverse.

Which of the statements given above is/are correct? A. 1 only B. 2 only C. Both 1 and 2 D. Neither 1 nor 2

ANS - A

RBI announced the Rs 50 denomination notes that have a motif of Hampi with Chariot on the reverse. Tourists can no longer get too close to the iconic stone chariot in front of the Vijaya Vittala Temple at Hampi, a UNESCO World Heritage site. The architectural marvel, one of the most photographed monuments have been cordoned off by a chain barricade by the Archaeological Survey of India (ASI).

Q) Which government entity banned the recruitment of Telephone Attendant cum Dak Khalasi (TADK), better known as 'Bungalow Peons'?

A. Indian Administrative Services B. Indian Railways C. Indian Judicial Services D. Indian army

ANS - B

The Indian Railways has banned the recruitment of Telephone Attendant cum Dak Khalasi (TADK), better known as ‘Bungalow Peons’. Curtains were drawn on the age-old tradition of allowing senior Railway officers to nominate a person to be appointed as peons to work in their residences or camp offices. Despite stiff resistance from Railways Officers’ Associations from across the country, the Ministry of Railways went ahead with its plan to end the practice, and also directed General Managers of Zonal Railways and other establishments to review existing posts

22 TELEGRAM LINK: https://t.me/opdemy WEBSITE: www.opdemy.com

Q) Consider the following statements: 1. Animal Adoption Programme at Bannerghatta Biological Park involves feeding and veterinary care expenses for zoo animals with a provision for an Income Tax rebate. 2. Bannerghatta Biological Park is in Madhya Pradesh.

Which of the statements given above is/are incorrect? A. 1 only B. 2 only C. Both 1 and 2 D. Neither 1 nor 2

ANS - B

Bannerghatta Biological Park is in Karnataka. Despite job losses and pay cuts across multiple sectors due to the pandemic, the Bannerghatta Biological Park’s (BBP) animal adoption programme received the highest ever response this year. The BBBP has released a list of animals that can be adopted: King cobra, Indian rock python, Black buck, Sambar, Emu, Golden jackal, Indian leopard, Sloth bear, Hippopotamus, Bengal tiger, Giraffe, etc.

Q) Consider the following statements: 1. Japan’s Hayabusa 2 space vehicle is set to return to Earth on December 6. 2. The space vehicle is carrying with it samples from the one-kilometer wide Ryugu asteroid that orbits the Sun.

Which of the statements given above is/are correct? A. 1 only B. 2 only C. Both 1 and 2 D. Neither 1 nor 2

ANS - C

Hayabusa2 is an asteroid sample-return mission operated by the Japanese space agency, JAXA. After dropping off its samples, Hayabusa2 will complete a series of orbits around the sun for around six years, recording data on dust in interplanetary space and observing exoplanets. It will then approach the first of its target asteroids in July 2026.

Q) Consider the following statements: 1. India’s first homes for transgender children in need of care and protection will soon be established in Bengaluru. 2. The project is being set up by the Ministry of Minorities Affairs.

23 TELEGRAM LINK: https://t.me/opdemy WEBSITE: www.opdemy.com

Which of the statements given above is/are incorrect? A. 1 only B. 2 only C. Both 1 and 2 D. Neither 1 nor 2

ANS - B

The Ministry of Women & Child Development is setting up this project.

Q) Vice President Venkaiah Naidu has virtually released a commemorative Postage Stamp in honor of which late Prime Minister of India?

A. B. Indira Gandhi C. IK Gujral D. Lal Bahadur Shashtri

ANS - C

I K Gujral was the 12th Prime Minister of India, he served as the Prime Minister from April 1997-March 1998. He was born in Jhelum (now in Pakistan) in 1919.

Q) West Bengal Chief Minister, Mamata Banerjee named the newly built “Majerhat bridge” in Kolkata as ‘Jai Hind’ bridge to commemorate the 125th Birth Anniversary of which Freedom Fighter?

A. Subhash Chandra Bose B. Baghajatin C. Bankim Chandra Chatterjee D. Jatindra Mohan Sengupta

ANS - A

Q) Consider the following statements: 1. Ranjitsinh Disale is the 1st Indian to win the 2020 Global Teacher Prize 2. He won the award in recognition of his contribution towards promoting education for disabled people.

Which of the statements given above is/are correct?

A. 1 only

24 TELEGRAM LINK: https://t.me/opdemy WEBSITE: www.opdemy.com

B. 2 only C. Both 1 and 2 D. Neither 1 nor 2

ANS - A

He won the award in recognition of his efforts to promote girls’ education in his village and trigger a quick-response (QR) coded textbook revolution in India. He made efforts and added QR codes to textbooks of primary classes to provide links to audio poems, video lectures, stories, and assignments. He also worked towards education for girls and teaching people about the ill-effects of teenage marriage and even helped to achieve 100% attendance of girls.

Q) Passage Exercise (PASSEX) in the Eastern Indian Ocean Region (IOR) is naval exercise between India & which country?

A. United States of America B. Japan C. Russia D. Australia

ANS - C

Q) With reference to Indus Valley Civilisation, consider the following statements: 1.Harappa is situated on the Bank of river Indus in Larkana district of Punjab (Pakistan). 2. Lipids are less prone to degradation and have been discovered in ceramic vessels at seven Indus Valley Civilisation sites in present-day Haryana and Uttar Pradesh.

Which of the statements given above is/are correct? A. 1 only B. 2 only C. Both 1 and 2 D. Neither 1 nor 2

ANS - B

Harappa is situated on the bank of river Ravi in Montgomery district of Punjab (Pakistan). Mohenjodaro (Mound of Dead) is situated on the bank of river Ravi in Montgomery district of Punjab (Pakistan).

Q) With reference to the Citizenship (Amendment) Act (CAA), consider the following

25 TELEGRAM LINK: https://t.me/opdemy WEBSITE: www.opdemy.com

statements: 1. In the Bill, Hindus, Sikhs, Buddhists, Jains, Parsis, and Christians from Afghanistan, Bangladesh, Myanmar, and Pakistan, who entered India on or before December 31, 2014, will not be treated as illegal migrants. 2. The Citizenship (Amendment) Act, 2019 seeks to amend the Citizenship Act, 1955.

Which of the statements given above is/are incorrect? A. 1 only B. 2 only C. Both 1 and 2 D. Neither 1 nor 2

ANS - A

The Bill amends the Act to provide that the Hindus, Sikhs, Buddhists, Jains, Parsis, and Christians from Afghanistan, Bangladesh and Pakistan, who entered India on or before December 31, 2014, will not be treated as illegal migrants.

Q) Which among the following is the world’s largest vaccine producer by volume?

A. AstraZeneca B. Pfizer C. Serum Institute of India D. Bharat Biotech

ANS - C

The SII is the world’s largest vaccine producer by volume and has become the second company to apply for the emergency authorization after Pfizer, India. Pfizer India had applied for emergency-use authorization for its COVID-19 vaccine, after the company’s parents received clearance for the treatment from Britain and Bahrain.

Q) Consider the following statements: 1. Assam’s Pobitora Wildlife Sanctuary is referred to as ‘Mini Kaziranga’. 2. Kaziranga hosts two-thirds of the world's great one-horned rhinoceroses, is a World Heritage Site.

Which of the statements given above is/are incorrect?

A. 1 only B. 2 only C. Both 1 and 2 D. Neither 1 nor 2

ANS - D

26 TELEGRAM LINK: https://t.me/opdemy WEBSITE: www.opdemy.com

Too many cattle are robbing the one-horned rhinos of Assam’s Pobitora Wildlife Sanctuary referred to as ‘Mini Kaziranga’ for similar features, of their nutritious food. One of the factors behind the crisis of nutritional grass in Pobitora is the competition from some 10,000 cattle that graze on the fringes of the sanctuary measuring 38.81 sq km on paper but with only 16 sq km for an estimated 102 rhinos to inhabit.

Q) With reference to World Heritage City, consider the following statements: 1. Gwalior and Orchha have been included in the list of World Heritage City by UNESCO. 2. UNESCO is headquartered in Geneva, Switzerland.

Which of the statements given above is/are correct?

A. 1 only B. 2 only C. Both 1 and 2 D. Neither 1 nor 2

ANS - A

UNESCO is headquartered in Paris, France. Orchha located in the Bundelkhand region of Madhya Pradesh was the 16th century capital of the erstwhile Bundela dynasty. At the same time, Gwalior city has been the capital of Gurjar Pratihar, Tomar, Baghel Kachhwahs, and Scindia Rajvansh.

Q) With reference to a report by Stockholm International Peace Research Institute (SIPRI), consider the following statements: 1. The U.S. arms industry accounted for 61% of sales by the world’s “Top 25” manufacturers last year 2. West Asia made its first appearance among the 25 biggest weapons Manufacturers.

Which of the statements given above is/are correct?

A. 1 only B. 2 only C. Both 1 and 2 D. Neither 1 nor 2

ANS - C

U.S. and Chinese companies dominated the global arms market in 2019, while West Asia made its first appearance among the 25 biggest weapons

27 TELEGRAM LINK: https://t.me/opdemy WEBSITE: www.opdemy.com

manufacturers, a report by the Stockholm International Peace Research Institute (SIPRI) said.

Q) India remains in the top 10 in Climate Change Performance Index (CCPI) which is released by?

A. World Economic Forum B. World Meteorological Organization C. World Bank D. Climate Action Network

ANS - D

The CCPI is developed by not-for-profit organizations Germanwatch and NewClimate Institute (Germany) together with the Climate Action Network (CAN International).

Q) What is the name of the new digital platform for COVID-19 vaccine delivery introduced by the Government of India?

A. CO-VAC B. CO-WIN C. CO-FREE D. CO-VAX

ANS - B

The Government of India has introduced a new digital platform called 'CO-WIN' for COVID-19 vaccine delivery. There will be a new mobile app as well with the same name that will allow people to register for the vaccine. The CO-WIN app is yet to be available for smartphone users and it is expected that it will be publicly available soon.

Q) Consider the following statements: 1. ‘Operation Blackface’ is a campaign against Child Sexual Abuse Material (CSAM). 2. The cyber wing of the Uttar Pradesh Police recently acquired software from Interpol would help them in this campaign.

Which of the statements given above is/are incorrect?

A. 1 only B. 2 only C. Both 1 and 2

28 TELEGRAM LINK: https://t.me/opdemy WEBSITE: www.opdemy.com

D. Neither 1 nor 2

ANS - B

The cyber wing of the Maharashtra Police recently acquired software from Interpol that would help them in this campaign. ‘Operation Blackface’ is part of the larger action taken against CSAM across the country. Acting on the tip-off provided by NCRB, Maharashtra Cyber cops started forwarding complaints to the districts where FIR’s were registered against accused persons.

Q) Which ministry released the Smuggling in India Report 2019-20 which analyses organized smuggling trends on Gold and Foreign Currency, Narcotic Drugs, Security, Environment, and Commercial Frauds?

A. Ministry of Electronics and Information Technology B. Ministry of Home Affairs C. Ministry of Communications and Information Technology D. Ministry of Finance

ANS - D

The report has been compiled by the Directorate of Revenue Intelligence (DRI).

Q) With reference to the Parliament of India, consider the following statements: 1.It was designed by Edwin Lutyens and Herbert Baker, who were responsible for planning and construction of New Delhi by the British government. 2. The opening ceremony was performed on 18 January 1927 by the then Viceroy and Governor-General of India, Lord Willingdon.

Which of the statements given above is/are correct?

A. 1 only B. 2 only C. Both 1 and 2 D. Neither 1 nor 2

ANS - A

The Parliament House (Sansad Bhavan) is located in New Delhi. It was designed by Edwin Lutyens and Herbert Baker, who were responsible for planning and construction of New Delhi by the British government. The construction of the building took six years and the opening ceremony was performed on 18 January 1927 by the then Viceroy and Governor-General

29 TELEGRAM LINK: https://t.me/opdemy WEBSITE: www.opdemy.com

of India, Lord Irwin.

Q) Consider the following statements: 1. HelpAge India has been presented the UN Population Award for 2020 (institutional category), according to a release issued by UNFPA. 2. For the first time in the history of the UN Population Award, the honour is being conferred on an Indian institution.

Which of the statements given above is/are correct? A. 1 only B. 2 only C. Both 1 and 2 D. Neither 1 nor 2

ANS - C

HelpAge India is an Indian organization focused on the concerns of elders. Established in 1978, its mission is “to work for the cause and care of disadvantaged older persons and to improve their quality of life”. The last time the Award came to an Indian was 28 years ago, back in 1992, when it was awarded to Mr. J.R.D. Tata as an individual laureate. HelpAge India, which has been working for ‘the cause and care of disadvantaged older persons to improve their quality of life’ for over four decades, is the first Indian institution to receive this award,”

Q) Who among the following has become the first person in the world to receive the Pfizer COVID-19 vaccine outside of a trial following its rapid clinical Approval?

A. Margaret Keenan B. Sacha Baron Cohen C. Hugh Grant D. Elizabeth Hurley

ANS - A

Margaret Keenan, a 90-year-old grandmother from Britain, has become the first person in the world to receive the Pfizer COVID-19 vaccine outside of a trial following its rapid clinical approval. Britain began rolling out the COVID-19 vaccine developed by Pfizer and BioNTech.

Q) Consider the following statements: 1. Morocco has become the fourth Arab nation this year to recognize Israel 2. The capital of Morocco is Tripoli.

30 TELEGRAM LINK: https://t.me/opdemy WEBSITE: www.opdemy.com

Which of the statements given above is/are incorrect?

A. 1 only B. 2 only C. Both 1 and 2 D. Neither 1 nor 2

ANS - B

The capital of Morocco is Rabat. Morocco has become the fourth Arab nation this year to recognize Israel, U.S. President Donald Trump announced Thursday, as he, in turn, fulfilled a decades-old goal of Morocco by backing its sovereignty in disputed Western Sahara.

Q) Consider the following statements: 1. Kuwait Emir Sheikh Nawaf Al-Ahmad Al-Sabah has re-appointed as the prime minister. 2. The currency of Kuwait is Riyal.

Which of the statements given above is/are correct?

A. 1 only B. 2 only C. Both 1 and 2 D. Neither 1 nor 2

ANS - A

The currency of Kuwait is Dinar. Kuwait's Emir Sheikh Nawaf al-Ahmad al-Sabah reappointed Sheikh Sabah al-Khalid al-Sabah as prime minister on Tuesday following parliamentary polls in the Gulf Arab state, which faces its worst economic crisis in decades.

31 TELEGRAM LINK: https://t.me/opdemy WEBSITE: www.opdemy.com

UPSC CSE PRELIMS 2021

CRASH COURSE DATE SUBJECT 10/04/2021, 11/04/2021 Budget and Economic Survey

12/04/2021, 13/04/2021 Art and Culture

14/0/2021, 15/04/2021, Economics 16/04/2021

17/18/2021,18/04/2021, Modern Indian History 19/04/2021

20/04/2021,21/04/2021, Geography 22/04/2021

23/04/2021,24/04/2021, Enviroment 25/04/2021

26/04/2021,27/04/2021, 28/04/2021,29/04/2021, Polity 30/04/2021

01/05/2021, 02/05/2021 Science and Tech *Note: Also, there will be Lectures on CSAT as and when time permits* PRICE: 1,999 For Any Queries: 7988797680, 7999136838, 6295118940

32 TELEGRAM LINK: https://t.me/opdemy WEBSITE: www.opdemy.com

Q) Consider the following statements: 1. Better Than Cash Alliance is a partnership of governments, companies and international organizations that accelerate the transition from cash to responsible digital payments. 2. With 75 members, the alliance is an organization of the World Bank.

Which of the statements given above is/are correct? A. 1 only B. 2 only C. Both 1 and 2 D. Neither 1 nor 2

ANS - A

The alliance (with 75 members) is a United Nations based alliance.

Q) Who has recently been appointed as the new President of International Golf Federation?

A. Peter Dawson B. Annika Sorenstam C. Zena Wooldridge D. Yulimar Rojas

ANS - B

Q)The World Economic Forum 2021 will be held in which country?

A. Beijing B. Paris C. Tokyo D. Singapore

ANS - D

The World Economic Forum will hold its annual meeting in Singapore in 2021, marking only the second time in two decades that the global gathering of business and political elites will have convened outside the Swiss ski town of Davos.

33 TELEGRAM LINK: https://t.me/opdemy WEBSITE: www.opdemy.com

Q)Consider the following statements: 1. Madagascar has replaced the Maldives as the host of the 2023 Indian Ocean Island Games. 2. The capital of Madagascar is Antananarivo.

Which of the statements given above is/are incorrect?

A. 1 only B. 2 only C. Both 1 and 2 D. Neither 1 nor 2

ANS - D

Madagascar has replaced the Maldives as the host of 2023 Indian Ocean Island Games due to concerns over the COVID-19 pandemic. The event had been awarded to the Maldives last year, but members of the Indian Ocean Island Games Federation voted to move the Games to Madagascar.

Q) According to the new 'Policy on School Bag 2020' school bags should not be more than how much percent of the bodyweight of students?

A. 20% B. 15% C. 10% D. 12%

ANS - C

School bags should not be more than 10% of the bodyweight of students across classes, I to X and there should be no homework till class II. The new 'Policy on School Bag 2020' of the Union ministry of education also recommends that the weight of the bag needs to be monitored on a regular basis in schools.

Q) With reference to the ISRO “frame-up” case of 1994, consider the following statements: 1.Nambi Narayanan is an Indian scientist and aerospace engineer and Padma Bhushan award winner. 2. In 1994, he was falsely charged with espionage and arrested.

Which of the statements given above is/are correct?

34 TELEGRAM LINK: https://t.me/opdemy WEBSITE: www.opdemy.com

A. 1 only B. 2 only C. Both 1 and 2 D. Neither 1 nor 2

ANS - C

Padma Bhushan. S. Nambi Narayanan (12 December 1941) is an Indian scientist and aerospace engineer and Padma Bhushan award winner. As a senior official at the Indian Space Research Organization (ISRO), he was in charge of the cryogenics division. In 1994, he was falsely charged with espionage and arrested.

Q) With reference to Delimitation Commission, consider the following statements: 1. Delimitation is conducted to have a fair division of geographical areas so that no political party has an undue advantage over the others. 2. As per a paper released by the Pranab Mukherjee Foundation (PMF), Delimitation Commission should redraw boundaries of constituencies on the basis of 2031 Census

Which of the statements given above is/are incorrect?

A. 1 only B. 2 only C. Both 1 and 2 D. Neither 1 nor 2

ANS - D

Delimitation is the process of fixing limits or boundaries of territorial constituencies in a country to represent changes in population. A paper released by the Pranab Mukherjee Foundation (PMF) on the eve of the late President’s birth anniversary on Friday suggested that the next delimitation exercise should be a two-step process: first, a Delimitation Commission should be set up to redraw boundaries of constituencies on the basis of the 2031 Census and then a State Reorganisation Act be passed to split States into smaller ones.

Q) The Shakti Act drafted to propose the death penalty for heinous crimes against women & children will be tabled in which State Assembly?

A. Rajasthan State Assembly B. Maharashtra State Assembly C. Kerala State Assembly D. Uttar Pradesh State Assembly

ANS - B

35 TELEGRAM LINK: https://t.me/opdemy WEBSITE: www.opdemy.com

In a bid to curb crimes against women and children in Maharashtra, the state cabinet approved two draft bills that propose the death penalty for heinous cases of rape, acid attack, and child abuse. The two bills that are the part of Shakti Act are the Maharashtra Shakti Criminal Law (Maharashtra Amendment) Act 2020 and the Special Court and Machinery for Implementation of Maharashtra Shakti Criminal Law 2020.

Q) Consider the following statements: 1. El Nino results in the decline of sea surface temperatures. 2. El Nino adversely impacts the Indian monsoons and hence, agriculture in India.

Which of the statements given above is/are correct?

A. 1 only B. 2 only C. Both 1 and 2 D. Neither 1 nor 2

ANS - B

El Nino results in the rise of sea surface temperatures It also weakens the trade winds of the affected region In India, Australia, it can bring about drought conditions. This affects crop productivity largely. It has been also observed certain times, that EL Nino may not bring drought but cause heavy rainfall. In both cases, it causes heavy damage.

Q) Consider the following statements: 1. The four-story new parliament building will be triangular in shape. 2. In the Lok Sabha chamber, the national bird (peacock) will be the theme & in the Rajya Sabha the national flower (lotus) will be the theme.

Which of the statements given above is/are correct? A. 1 only B. 2 only C. Both 1 and 2 D. Neither 1 nor 2

ANS - C

The four-story building will be triangular and its interiors will have three

36 TELEGRAM LINK: https://t.me/opdemy WEBSITE: www.opdemy.com

national symbols - the lotus, the peacock, and the banyan tree - as its themes. The triangular shape of the new parliament was a reference to "sacred geometries in various religions and cultures of India". In the Lok Sabha chamber, the national bird (peacock) will be the theme. In the Rajya Sabha, the national flower (lotus) and in the central lounge the national tree (banyan) will be the theme.

Q) Consider the following statements: 1. The International Olympic Committee (IOC) has registered ‘Breakdancing’ as the newest entry into the Olympics and an official Olympic sport. 2. It will make its Olympic Games debut at the Paris Games 2024 Summer Olympics.

Which of the statements given above is/are correct?

A. 1 only B. 2 only C. Both 1 and 2 D. Neither 1 nor 2

ANS - C

Q) The National Centre for Divyang Empowerment has recently been inaugurated by the Home Minister of which State?

A. Maharashtra B. Uttar Pradesh C. Jharkhand D. Telangana

ANS - D

The Union Minister of State for Home Affairs, G Kishan Reddy has inaugurated the National Centre for Divyang Empowerment (NCDE) at the CRPF Group Centre in Hakimpet near Hyderabad, Telangana.

Q) Who has been awarded the 2020 Ramanujan Prize for Young Mathematicians?

A. Carolina Araujo B. Nalini Anantharaman C. Marcelo Viana D. Jacob Pallis

ANS - A

37 TELEGRAM LINK: https://t.me/opdemy WEBSITE: www.opdemy.com

Dr. Carolina Araujo, Mathematician from the Institute for Pure and Applied Mathematics (IMPA), Rio de Janeiro, Brazil was awarded the 2020 Ramanujan Prize for Young Mathematicians. Her work area focuses on birational geometry, which aims to classify and describe the structure of algebraic varieties.

Q) Consider the following statements: 1. The Union Minister for Road Transport and Highways, Nitin Gadkari has recently inaugurated a 1.5 km long Koilwar bridge over the Sone river in Uttar Pradesh. 2. This newly inaugurated bridge will a part of the six-lane bridge being made instead of the existing two-lane bridge, for both rail and road traffic, which is nearly 138 years old.

Which of the statements given above is/are incorrect?

A. 1 only B. 2 only C. Both 1 and 2 D. Neither 1 nor 2

ANS - B

The bridge has been inaugurated in Bihar. Though it will connect UP & Bihar, which will reduce the traffic of NH-30 & NH-922.

Q) Recently, Prime Minister greeted the people of which of the following countries on the festival of Chag Hanukkah Sameach?

A. Sri Lanka B. Australia C. Qatar D. Israel

ANS - D

Prime Minister Narendra Modi greeted the people of Israel on the festival of Chag Hanukkah Sameach. Hanukkah is also known as Chanukah. It is an eight-day Jewish celebration that marks the rededication during the second century BC of the Second Temple in Jerusalem.

Q) With reference to Kaziranga National Park, consider the following statements: 1.Boats and bicycles are also finalized as modes of regulated transport to call on the rhinos in KNP.

38 TELEGRAM LINK: https://t.me/opdemy WEBSITE: www.opdemy.com

2. It is the single largest undisturbed and representative area in the Brahmaputra Valley floodplain.

Which of the statements given above is/are correct?

A. 1 only B. 2 only C. Both 1 and 2 D. Neither 1 nor 2

ANS - C

Elephants and jeeps are no longer the only modes of regulated transport to call on the rhino in the Kaziranga National Park and Tiger Reserve billed as the best-conserved home of the one-horned animal. In November, the park authorities launched a boat safari at Bhomoraguri after the addition of 3,053 hectares to the park. Bhomoraguri is about 30 km from Kaziranga’s westernmost range Burapahar.

Q) With reference to dry state in India, consider the following statements: 1. Alcohol prohibition in India is related to directive principles of state policy (DPSP) in Article 47 in the . 2. Puducherry is the only Union Territory in India to ban the sale and consumption of alcohol.

Which of the statements given above is/are incorrect? A. 1 only B. 2 only C. Both 1 and 2 D. Neither 1 nor 2

ANS - B

Lakshadweep is the only Union Territory in India to ban the sale and consumption of alcohol. The latest National Family Health Survey-5 (NFHS-5) report for 2019-20 which was released on Saturday (December 12) has revealed that still 15.5% of men above 15 years of age consume alcohol in dry Bihar.

Q) Consider the following statements: 1. India’s nursing workforce is about two-thirds of its health workforce which is 43% less than the World Health Organisation norm 2. The year 2021 has been designated as “International Year of the Nurse and the Midwife”

39 TELEGRAM LINK: https://t.me/opdemy WEBSITE: www.opdemy.com

Which of the statements given above is/are correct?

A. 1 only B. 2 only C. Both 1 and 2 D. Neither 1 nor 2

ANS - A

The year 2020 has been designated as “International Year of the Nurse and the Midwife”. Nurses and midwives will be central to achieving universal health coverage in India. India’s nursing workforce is about two-thirds of its health workforce. Its ratio of 1.7 nurses per 1,000 population are 43% less than the World Health Organization norm; it needs 2.4 million nurses to meet the norm.

Q) Consider the following statements: 1. The Climate Ambition Summit, held virtually, marked the five years of the adoption of the Paris Agreement on Climate Change. 2. India has set an even more ambitious target of 450 GigaWatts of renewable energy capacity by 2030.

Which of the statements given above is/are correct?

A. 1 only B. 2 only C. Both 1 and 2 D. Neither 1 nor 2

ANS - C

India hosted the Climate Ambition Summit 2020, held virtually, on December 12. The United Nations, United Kingdom, and France co-hosted the Climate Ambition Summit 2020, held virtually, on December 12. The Climate Ambition Summit, held virtually, marked the five years of the adoption of the Paris Agreement on Climate Change. India has reduced emission intensity by 21% over 2005 levels. Solar capacity has grown from 2.63 GigaWatts in 2014 to 36 GigaWatts in 2020. Renewable energy capacity is the fourth largest in the world. It will reach 175 GigaWatts before 2022. India has set an even more ambitious target now - 450 GigaWatts of renewable energy capacity by 2030.

Q) Consider the following statements: 1. Chief of Army Staff Manoj Mukund Narvane is on a six-day visit to Saudi

40 TELEGRAM LINK: https://t.me/opdemy WEBSITE: www.opdemy.com

Arabia & UAE. 2. India & UAE have been conducting a Joint Air Force exercise named "Gulf Star I".

Which of the statements given above is/are correct?

A. 1 only B. 2 only C. Both 1 and 2 D. Neither 1 nor 2

ANS - A

The Joint Air Force exercise between India & UAE is called "Desert Eagle". The name "Gulf Star I" is of the Joint Naval Exercise between India & UAE.

Q) Consider the following statements: 1. Maharashtra's state police cybercrime prevention department of Cyber Unit has acquired Interpol's "Crawler" software to track child porn. 2. The whole mission has been named "Operation Blackface" to curb pedophilia.

Which of the statements given above is/are correct?

A. 1 only B. 2 only C. Both 1 and 2 D. Neither 1 nor 2

ANS - C

Q) The Ministry of Agriculture has declared which Union Territory as India’s first UT to become 100 percent organic in terms of farming activities?

A. Lakshadweep B. Chandigarh C. Puducherry D. Daman & Diu

ANS - A

Lakshadweep is second only to Sikkim, which was India’s first state to be declared completely organic.

Q) Which Indian Telecom company has launched the World’s 1st Satellite-based Narrowband-IoT Network?

41 TELEGRAM LINK: https://t.me/opdemy WEBSITE: www.opdemy.com

A. Reliance B. BSNL C. Idea Network D. Vodafone

ANS - B

BSNL launches 'world's first' satellite-based narrowband-IoT network. BSNL on Thursday announced the launch of a satellite-based internet of things (IoT) device service that can be used across India where mobile towers are not present, including the seas within the country's jurisdiction

Q) Consider the following statements: 1. Bajrang Punia & Elavenil Valarivan have been awarded "Sportsperson of the Year 2020" by the India Sports Award 2020 2. Bajrang Punia belongs to wrestling sport and Elavenil Valarivan belongs to the 100m sprint race.

Which of the statements given above is/are incorrect?

A. 1 only B. 2 only C. Both 1 and 2 D. Neither 1 nor 2

ANS - B

Bajrang Punia belongs to the wrestling sport & Elavenil Valarivan belongs to the shooting sport.

Q) Which bank has recently sanctioned USD 9 million for an initiative called the Asia Pacific Vaccine Access Facility (APVAX), for its developing member countries?

A. World Bank B. International Monetary Fund C. Asian Development Bank D. New Development Bank

ANS - C

ADB has launched a USD 9-billion vaccine initiative, the Asia-Pacific Vaccine Access Facility (APVAX), offering support to its developing members as they

42 TELEGRAM LINK: https://t.me/opdemy WEBSITE: www.opdemy.com

procure and deliver vaccines for COVID-19, it said in a release.

Q) Consider the following statements: 1.PM Narendra Modi will lay the foundation stone for World’s largest hybrid renewable energy park in Gujarat’s Narmada district. 2. The park will generate 30,000 MW power through solar panels and windmills on 72,600 hectares along the India-Pakistan border.

Which of the statements given above is/are correct?

A. 1 only B. 2 only C. Both 1 and 2 D. Neither 1 nor 2

ANS - B

Prime Minister Narendra Modi on Tuesday will lay the foundation stone for what is billed as the world’s largest hybrid renewable energy park in Gujarat’s Kutch district.

Q) With reference to Real-Time Gross Settlement (RTGS), consider the following statements: 1. The minimum amount needed to be transferred has to be Rs. 1 Lakhs and above for RTGS 2. The Real-Time Gross Settlement system was first implemented in India in March 2004 as a major technology-based electronic funds transfer system

Which of the statements given above is/are incorrect? A. 1 only B. 2 only C. Both 1 and 2 D. Neither 1 nor 2

ANS - A

The minimum amount needed to be transferred has to be Rs. 2 Lakhs and above for RTGS

Q) Which organization was awarded ‘The Most Innovative Research Institute of the Year’ under Innovation Quotient in the CII Industrial Innovation Awards 2020?

A. IIT Madras

43 TELEGRAM LINK: https://t.me/opdemy WEBSITE: www.opdemy.com

B. IIT Roorkee C. IIT Kanpur D. IIT Delhi

ANS - B

CII Industrial Innovation Awards 2020 were presented during the 26th DST-CII Technology Summit 2020. 18 regular awards, 8 category awards, and 1 grand award were given to the most innovative company of the year. IIT Roorkee adjudged as ‘The Most Innovative Research Institute of the Year’ under Innovation Quotient. IIT Madras adjudged as ‘The Most Innovative Research Institute of the Year’ under Disruptive Innovations.

Q) With reference to National Family Health Survey-5 (NFHS-5), consider the following statements: 1. Kerala and Karnataka are the only two big States among the six States and UTs where there is some decline in malnutrition. 2. The World Health Organization calls stunting “a marker of inequalities in human Development”.

Which of the statements given above is/are correct?

A. 1 only B. 2 only C. Both 1 and 2 D. Neither 1 nor 2

ANS - C

Of the 22 States and UTs, there is an increase in the prevalence of severe acute malnutrition in 16 States/UTs (compared to NFHS-4 conducted in 2015-16). Kerala and Karnataka are the only two big States among the six States and UTs where there is some decline. The percentage of children under five who are underweight has also increased in 16 out of the 22 States/UTs. Anaemia levels among children as well as adult women have increased in most of the States with a decline in anaemia among children being seen only in four States/UTs (all of the smaller ones — Lakshadweep, Andaman and Nicobar Islands, Dadra and Nagar Haveli and Daman and Diu, and Meghalaya).

Q) Consider the following statements:

44 TELEGRAM LINK: https://t.me/opdemy WEBSITE: www.opdemy.com

1. Himgiri, the first of the three stealth frigates under Project 17A for the Navy was launched into water on Monday. 2. Himgiri being built by Larsen & Toubro

Which of the statements given above is/are correct?

A. 1 only B. 2 only C. Both 1 and 2

D. Neither 1 nor 2

ANS - A

Himgiri, the first of the three stealth frigates being built by Garden Reach Shipbuilders and Engineers (GRSE), Kolkata, under Project 17A for the Navy, was launched into water on Monday. In Goa, the fifth and last of the offshore patrol vessels (OPV) in the series Saksham being built at Goa Shipyard was launched into the water and is expected to be delivered to the Indian Coast Guard (ICG) by October 2021.

Q) Consider the following statements: 1. UNESCO has decided to launch an international prize in the field of ‘creative economy’ in the name of Bangabandhu Sheikh Mujibur Rahman. 2. Sheikh Mujibur Rahman served as the first President of Bangladesh

Which of the statements given above is/are correct?

A. 1 only B. 2 only C. Both 1 and 2 D. Neither 1 nor 2

ANS - C

Q) Myristica swamp treefrog is a rare arboreal species endemic to:

A. Eastern Ghats B. Western Himalayas C. Northeast Himalayas D. Western Ghats

ANS - D

45 TELEGRAM LINK: https://t.me/opdemy WEBSITE: www.opdemy.com

Myristica swamp treefrog, a rare arboreal species endemic to the Western Ghats has been recorded for the first time north of the Shencottah gap in the Vazhachal Reserve Forest in Kerala’s Thrissur district. Its scientific name is Mercurana myristicapalustris. These frogs are rare and elusive for the reason that they are arboreal and active only for a few weeks during their breeding season.

Q) ‘Swadhinata Sarak’ recently in news is between India and which of the following neighboring countries?

A. Nepal B. Myanmar C. Bhutan D. Bangladesh

ANS - D

Bangladesh Foreign Minister has said that the ‘Swadhinata Sarak’ between Bangladesh and India will be opened on 26 March next year. The road remains functional on the Indian side while it will be connected through Mujibnagar, Meherpur district in Bangladesh.

Q) Consider the following statements: 1. Vidyavachaspati Bannanje Govindacharya was a Sanskrit and Kannada scholar. 2. He was honored with Padma Shri, India's fourth-highest civilian honor in 2009.

Which of the statements given above is/are incorrect?

A. 1 only B. 2 only C. Both 1 and 2 D. Neither 1 nor 2

ANS - D

Sanskrit and Kannada scholar Vidyavachaspati Bannanje Govindacharya, 84, passed away due to age-related ailments.

Q) Who was the most searched personality on Google this year, as per Google India?

46 TELEGRAM LINK: https://t.me/opdemy WEBSITE: www.opdemy.com

A. Narendra Modi B. Joe Biden C. Sushant Singh Rajput D. Kangana Ranaut

ANS - B

As per Google India's Year in Search 2020 the result, Kanika Kapoor was the most searched personality after US President-elect Joe Biden and journalist Arnab Goswami. Amitabh Bachchan is on the fifth spot on the list followed by Rhea Chakraborty.

Q) With reference to Minimum Support Price (MSP), consider the following statements: 1.The MSP is fixed on the recommendations of the Commission for Agricultural Costs and Prices (CACP). 2. The CACP is a non-statutory body under the Ministry of Agriculture and Farmers Welfare.

Which of the statements given above is/are correct?

A. 1 only B. 2 only C. Both 1 and 2 D. Neither 1 nor 2

ANS - A

The CACP is an attached office of the Ministry of Agriculture and Farmers Welfare, formed in 1965. It is a statutory body that submits separate reports recommending prices for Kharif and Rabi seasons.

Q) With reference to PM WANI, consider the following statements: 1. PM WANI stands for Prime Minister Wi-Fi Access Network Interface Scheme 2. PDO will be set up on the lines of Public Call Offices (PCOs) that will involve licensing and registration.

Which of the statements given above is/are incorrect?

A. 1 only B. 2 only C. Both 1 and 2 D. Neither 1 nor 2

47 TELEGRAM LINK: https://t.me/opdemy WEBSITE: www.opdemy.com

ANS - B

The scheme envisages setting up public Wi-Fi networks and access points by local Kirana and neighborhood shops through public data offices (PDO will be set up on the lines of Public Call Offices (PCOs)) that will not involve any license, fee or registration. Apart from, Public Wi-Fi being a low-cost option to reach unserved citizens and grow the economy, it can revolutionize the tech world and significantly improve Wi-Fi availability across the length and the breadth of India.

Q) Ghana, officially the Republic of Ghana, is a country along which of the following the Gulf of Africa?

A. Gulf of Tadjoura B. Gulf of Guinea C. Gulf of Aqaba D. Gulf of Suez

ANS - B

Ghana, officially the Republic of Ghana, is a country along the Gulf of Guinea and the Atlantic Ocean, in the subregion of West Africa. Ghana is bordered by the Ivory Coast in the west, Burkina Faso in the north, Togo in the east, the Gulf of Guinea, and the Atlantic Ocean in the south. Ghana means "Warrior King" in the Soninke language.

Q) With reference to the Vision 2035: Public Health Surveillance in India, consider the following statements: 1. It was released by the World Economic Forum. 2. It lays out India’s vision 2035 for public health surveillance through the integration of the three-tiered public health system into Ayushman Bharat.

Which of the statements given above is/are correct?

A. 1 only B. 2 only C. Both 1 and 2 D. Neither 1 nor 2

ANS - B

NITI Aayog today released a white paper: Vision 2035: Public Health

48 TELEGRAM LINK: https://t.me/opdemy WEBSITE: www.opdemy.com

Surveillance in India. Its vision is: To make India’s public health surveillance system more responsive and predictive to enhance preparedness for action at all levels. A citizen-friendly public health surveillance system will ensure individual privacy and confidentiality enabled with a client feedback mechanism.

Q) Consider the following statements: 1. British Prime Minister Boris Johnson will be the chief guest of India’s Republic Day 2021 celebrations 2. The first Republic Day parade was held in 1950, and it has been held every year since.

Which of the statements given above is/are correct?

A. 1 only B. 2 only C. Both 1 and 2 D. Neither 1 nor 2

ANS - C

British Prime Minister Boris Johnson will be the chief guest of India’s Republic Day celebrations, U.K. Foreign Secretary Dominic Raab said here on Tuesday. He stressed that the Indo-Pacific region was one of the “highest” focus areas of the U.K.’s foreign policy.

Q) Consider the following statements: 1. India & Austria have recently signed an MoU on Technology Cooperation in the Road Infrastructure Sector. 2. The capital of Austria is Vienna.

Which of the statements given above is/are correct?

A. 1 only B. 2 only C. Both 1 and 2 D. Neither 1 nor 2

ANS - C

Q) Which landlocked country is seeking access to the Indian Chabahar Port in Iran & has recently held the first Trilateral Working Group (TWG) Meeting with India & Iran?

49 TELEGRAM LINK: https://t.me/opdemy WEBSITE: www.opdemy.com

A. Kazakhstan B. Armenia C. Kyrgyzstan D. Uzbekistan

ANS - D

Chabahar Port is a seaport in Chabahar located in south-eastern Iran, on the Gulf of Oman. It serves as Iran's only oceanic port, and consists of two separate ports named Shahid Kalantari and Shahid Beheshti.

Q) The International Hockey Federation (FIH) had announced that 2023 FIH Men’s Hockey World Cup will be hosted in which Indian State? A. Maharashtra B. Uttar Pradesh C. Odisha D. Tamil Nadu

ANS - C

This will be the 2nd consecutive time it will be held in Odisha. Bhubaneswar & Rourkela will be the main venues.

Q) Consider the following statements: 1. Ola has signed an MoU with the Maharashtra Govt to build the largest scooter manufacturing facility of the world. 2. Initially, the factory will have an annual production capacity of 2 million Units.

Which of the statements given above is/are incorrect?

A. 1 only B. 2 only C. Both 1 and 2 D. Neither 1 nor 2

ANS - A

Ola has signed the MoU with the Tamil Nadu Govt.

Q) Who has recently been declared as the winner of the Social Entrepreneur of the Year (SEOY) Award – India 2020?

50 TELEGRAM LINK: https://t.me/opdemy WEBSITE: www.opdemy.com

A. Ashraf Patel B. Shiv Nadar C. Dilip Sanghvi D. Azim Premji

ANS - A

The award was presented by the Union Minister of Women & Child Development & Textiles, Smriti Zubin Irani.

Q) With reference to ‘Covaxin’, consider the following statements: 1.‘Covaxin’ is the indigenous Covid-19 vaccine developed by Hyderabad-based Bharat Biotech. 2. It was developed from a strain of SARS-CoV-2 isolated at the National Institute of Virology.

Which of the statements given above is/are correct?

A. 1 only B. 2 only C. Both 1 and 2 D. Neither 1 nor 2

ANS - C

Phase-1 results show that Covaxin, a COVID-19 vaccine candidate, is safe. It was well-tolerated in all dose groups and generated a “robust immune response”, according to the results released in a preprint paper, titled ‘A Phase 1: Safety and Immunogenicity Trial of an Inactivated SARS-CoV-2 Vaccine - BBV152’.

Q) With reference to the Right to privacy, consider the following statements: 1. Right to privacy is an intrinsic part of life and liberty under Article 19. 2. Supreme Court in Justice K. S. Puttaswamy (Retd) Vs Union of India unanimously held that privacy is a natural right.

Which of the statements given above is/are correct?

A. 1 only B. 2 only C. Both 1 and 2 D. Neither 1 nor 2

ANS - B

In August 2017, a nine-judge bench of the Supreme Court in Justice K.

51 TELEGRAM LINK: https://t.me/opdemy WEBSITE: www.opdemy.com

S. Puttaswamy (Retd) Vs Union of India unanimously held that Indians have a constitutionally protected fundamental right to privacy that is an intrinsic part of life and liberty under Article 21.

Q) Which state is set to be the first state in India to have a dedicated action plan for the conservation of pangolins – the world's most trafficked animal? A. Kerala B. Chhattisgarh C. Maharashtra D. Andhra Pradesh

ANS - C

The Maharashtra forest department is set to be the first state in India to have a dedicated action plan for conservation of pangolins – the world's most trafficked animals. Pangolins are listed in Schedule I of India's Wildlife (Protection) Act, 1972, implying the highest degree of protection.

Q) Consider the following statements: 1. Bangladesh has recently signed an agreement with the Serum Institute of India for vaccine delivery. 2. 3 crore doses of COVAXIN will be delivered to Bangladesh under this agreement.

Which of the statements given above is/are correct?

A. 1 only B. 2 only C. Both 1 and 2 D. Neither 1 nor 2

ANS - A

3 crore doses of UK's Oxford/AstraZeneca vaccines will be developed when India will mass-produce the UK's vaccine formula.

Q) With reference to National Family Health Survey 5, consider the following statements: 1. Bihar has the highest percentage of stunted children (42.9%) among big States. 2. POSHAN Abhiyaan need to meet a 2%-point decrease in childhood stunting every year.

Which of the statements given above is/are correct?

52 TELEGRAM LINK: https://t.me/opdemy WEBSITE: www.opdemy.com

A. 1 only B. 2 only C. Both 1 and 2 D. Neither 1 nor 2

ANS - C

In the key indicator of childhood stunting, there has been an increase in 13 of the 22 States/UTs in comparison to NFHS-4, with a noticeable improvement only in Bihar and Assam. Even here, the reduction in stunting numbers is lower than the government’s targets. Bihar has shown a promising decline of 5.4 % points in stunting, but still retains the highest percentage of stunted children (42.9%) among big States.

Q) Consider the following statements: 1. The 47th G7 summit in 2021 will be held in the UK. 2. G10 includes the members of G7 along with India, South Korea & Australia.

Which of the statements given above is/are correct?

A. 1 only B. 2 only C. Both 1 and 2 D. Neither 1 nor 2

ANS - C

Q) Which country has signed an MoU with India to cooperate in the Clean Ganga Mission?

A. Sweden B. Norway C. France D. Germany

ANS - B

The Norwegian Institute of Bioeconomy Research (NIBIO) has signed an MoU with cGanga, a think-tank of National Mission for Clean Ganga (NMCG) for development of sludge management framework in India. Through this initiative, Norway looks forward to strengthen the bilateral ties with India especially in the fields of prevention of climate change & conservation of the environment through

53 TELEGRAM LINK: https://t.me/opdemy WEBSITE: www.opdemy.com

environment-friendly & energy-efficient means.

Q) After the recent removal of Sudan from the US list of State Sponsors of Terrorism, only three countries are now left in the lift. Which among the following countries are not on that list?

A. Syria B. Iran C. North Korea D. Somalia

ANS - D

Q) Consider the following statements & identify the country that is recently in the news. 1. It is making headlines because of the Tigray conflict. 2. Its capital houses the headquarter of the African Union

A. Egypt B. Algeria C. Ethiopia D. Democratic Republic of Congo

ANS - C

Q) S-400 air defense systems, often mentioned in the news, was developed in which of the following countries?

A. Russia B. U.S.A C. Turkey D. France

ANS - A

The United States has imposed sanctions on Turkey on Monday over its acquisition of Russian S-400 air defense systems. The S-400 Triumf, (NATO calls it SA-21 Growler), is a mobile, surface-to-air missile system (SAM) designed by Russia. It is the most dangerous operationally deployed modern long-range SAM (MLR SAM) in the world, considered much ahead of the US-developed Terminal High Altitude Area Defense system (THAAD).

54 TELEGRAM LINK: https://t.me/opdemy WEBSITE: www.opdemy.com

Q) With reference to the Right to Protest, consider the following statements: 1. Constitution guarantees the right to protest and express dissent, but with an obligation towards certain duties. 2. Right to protest and right to assemble peacefully without arms is a part under Article 21.

Which of the statements given above is/are correct?

A. 1 only B. 2 only C. Both 1 and 2 D. Neither 1 nor 2

ANS - A

The Constitution guarantees the right to protest and express dissent, but with an obligation towards certain duties. Article 19 confers upon citizens the right to freedom of speech and expression under Article 19(1)(a) and the right to assemble peacefully without arms under Article 19(1)(b). These rights, in cohesion, enable every citizen to assemble peacefully and protest against action or inaction of the State.

Q) Consider the following statements: 1. Chattogram and Mongla ports in Bangladesh are for the movement of goods to and from India. 2. The Feni river forms part of the India-Bangladesh border which originates in the South Assam.

Which of the statements given above is/are incorrect?

A. 1 only B. 2 only C. Both 1 and 2 D. Neither 1 nor 2

ANS - B

The Feni river forms part of the India-Bangladesh border. It originates in the South Tripura district, passes through Sabroom town on the Indian side, and meets the Bay of Bengal after it flows into Bangladesh.

Q) Consider the following statements: 1. Visakhapatnam is to be considered for being the executive capital. 2. Amaravati and Kurnool are to be the Judicial and Legislative capital respectively.

55 TELEGRAM LINK: https://t.me/opdemy WEBSITE: www.opdemy.com

Which of the statements given above is/are correct?

A. 1 only B. 2 only C. Both 1 and 2 D. Neither 1 nor 2

ANS - A

The state can have three capitals — legislative, executive, and judicial. Visakhapatnam, which has the infrastructure, can be considered for being the executive capital without much investment. Amaravati can be retained as the legislative capital. Kurnool can be considered as the judicial capital.

Q) Consider the following statements: 1. National Hydrology Project (NHP) was started in the year 2016 as a centrally sponsored Scheme. 2. Under the NHP, a nationwide repository of water resources data - NWIC has been established.

Which of the statements given above is/are correct? A. 1 only B. 2 only C. Both 1 and 2 D. Neither 1 nor 2

ANS - B

Review of the National Hydrology Project (World Bank-supported initiative Ministry of Jal Shakti) was carried out by the Minister of Jal Shakti. National Hydrology Project (NHP) was started in the year 2016 as a Central Sector Scheme with 100% grant to Implementing agencies on pan India basis with a budget outlay of Rs 3680 Crore to be spent over a period of 8 years. The project aims at improving the extent, reliability, and accessibility of water resources information and to strengthen the capacity of targeted water resource management institutions in India.

Q) Consider the following statements: 1. India's rank in the latest HDI ranking is 129th, according to the Human Development Report (HDR) 2020 released by the United Nations Development Program (UNDP). 2. India's rank would be 123rd if we include the Planetary Pressure adjusted HDI or PHDI.

56 TELEGRAM LINK: https://t.me/opdemy WEBSITE: www.opdemy.com

Which of the statements given above is/are correct?

A. 1 only B. 2 only C. Both 1 and 2 D. Neither 1 nor 2

ANS - B

India's rank in the latest HDI ranking is 131st. However, India's rank would improve 8 places to the 123rd spot if we include the Planetary Pressure adjusted HDI or PHDI. Norway has topped the list. Niger came last in the list of 189 countries.

Q) Consider the following statements:

1. The Indian Government & the World Bank has signed a $400 million project to support India’s efforts at providing social & economic assistance to the poor & vulnerable households that were severely impacted by the COVID-19 pandemic. 2. This is the first economic aid that India has received from the World Bank for tackling COVID -19 pandemic.

Which of the statements given above is/are correct?

A. 1 only B. 2 only C. Both 1 and 2 D. Neither 1 nor 2

ANS - A

This is the second project that India & the World Bank have decided to work on tackling COVID-19. The first operation was $750 million & was launched in May 2020. The $400 million credit is from the International Development Association (IDA) – the World Bank’s concessionary lending arm.

Q) The Government of India has signed a loan agreement for $1,000 million with which bank to provide support to Aatma Nirbhar Bharat Abhiyan through National Rural Employment Guarantee Scheme (MGNREGS)?

A. World Bank B. International Monetary Fund C. Asian Development Bank

57 TELEGRAM LINK: https://t.me/opdemy WEBSITE: www.opdemy.com

D. New Development Bank

ANS - D

Important points about the New Development Bank are as follows - ● Headquarters location: Shanghai, China ● President: Marcos Prado Troyjo ● Founded: 15 July 2014, Fortaleza, State of Ceará, Brazil ● Parent organization: BRICS

Q) The Union Cabinet has approved MoU between India and which country for exchange of information in areas of mutual interest in the electricity sector?

A. Norway B. Bangladesh C. USA D. Russia

ANS - C

The MoU will help in improving regulatory and policy framework for developing an efficient wholesale power market and enhancing grid reliability. Under the MoU, the two sides will identify energy-related issues and develop topics and possible agendas for the exchange of information and regulatory practices in areas of mutual interest.

Q) Consider the following statements: 1. The Union Cabinet has approved the Revised Cost Estimate (RCE) of North Eastern Region Power System Improvement Project (NERPSIP) at an estimated cost of Rs. 6,700 crore. 2. The scheme is set to be commissioned by December 2021. Once commissioned, the the project will be owned and maintained by the respective North-Eastern State Utilities.

Which of the statements given above is/are correct?

A. 1 only B. 2 only C. Both 1 and 2 D. Neither 1 nor 2

ANS - C

58 TELEGRAM LINK: https://t.me/opdemy WEBSITE: www.opdemy.com

The main aim of the project is the economic development of North Eastern Region through the strengthening of Intra - State Transmission and Distribution systems.

Q) Which Indian-American has been selected by the NASA & European Space Agency to be the Commander of the SpaceX Crew-3 mission to the International Space Station?

A. Jasmine Moghbeli B. Kayla Barron C. Sunita Williams D. Raja Chari,

ANS - D

Presently Raja Chari is a colonel in the US Air Force. He will be the commander of SpaceX Crew-3 mission, while Nasa’s Tom Marshburn will be pilot and ESA’s Matthias Maurer will serve as a mission specialist. This mission is expected to be launched next year.

Q) With reference to Minimum Support Price, consider the following statements: 1. MSP is price fixed by the Government on the recommendations of the Commission for Agricultural Costs and Prices (CACP). 2. Commission for Agricultural Costs and Prices is an attached office of the Ministry of Agriculture & Farmers Welfare.

Which of the statements given above is/are correct?

A. 1 only B. 2 only C. Both 1 and 2 D. Neither 1 nor 2

ANS - C

Minimum Support Price (MSP) is a form of market intervention by the Government of India to insure agricultural producers against any sharp fall in farm prices. The minimum support prices are announced by the Government of India at the beginning of the sowing season for certain crops on the basis of the recommendations of the Commission for Agricultural Costs and Prices (CACP). MSP is price fixed by the Government of India to protect the producer - farmers - against excessive fall in price during bumper production years. The minimum support prices are a guaranteed price for their produce from the Government.

59 TELEGRAM LINK: https://t.me/opdemy WEBSITE: www.opdemy.com

UPSC CSE PRELIMS 2021

CRASH COURSE DATE SUBJECT 10/04/2021, 11/04/2021 Budget and Economic Survey

12/04/2021, 13/04/2021 Art and Culture

14/0/2021, 15/04/2021, Economics 16/04/2021

17/18/2021,18/04/2021, Modern Indian History 19/04/2021

20/04/2021,21/04/2021, Geography 22/04/2021

23/04/2021,24/04/2021, Enviroment 25/04/2021

26/04/2021,27/04/2021, 28/04/2021,29/04/2021, Polity 30/04/2021

01/05/2021, 02/05/2021 Science and Tech *Note: Also, there will be Lectures on CSAT as and when time permits* PRICE: 1,999 For Any Queries: 7988797680, 7999136838, 6295118940

60 TELEGRAM LINK: https://t.me/opdemy WEBSITE: www.opdemy.com

Q) With reference to Contempt of Court, consider the following statements: 1. The objective for contempt is stated to be to safeguard the interests of the public if the authority of the Court is denigrated 2. The Supreme Court and High Courts derive their contempt powers from the Constitution.

Which of the statements given above is/are incorrect?

A. 1 only B. 2 only C. Both 1 and 2 D. Neither 1 nor 2

ANS - D

Contempt of Court refers to the offense of showing disrespect to the dignity or authority of a court. The objective for contempt is stated to be to safeguard the interests of the public, if the authority of the Court is denigrated and public confidence in the administration of justice is weakened or eroded. The Supreme Court and High Courts derive their contempt powers from the Constitution. The Contempt of Court Act, 1971, outlines the procedure in relation to investigation and punishment for contempt.

Q) Consider the following statements: 1. The Railways have so far incurred an 87% loss in passenger revenue, down from ₹53,000 crore last year to ₹4,600 crores, owing to COVID-19 2. India’s first passenger train, operated by the Great Indian Peninsula Railway ran between Bori Bunder (Mumbai) and Nasik in 1853

Which of the statements given above is/are correct?

A. 1 only B. 2 only C. Both 1 and 2 D. Neither 1 nor 2

ANS - A

India’s first passenger train, operated by the Great Indian Peninsula

61 TELEGRAM LINK: https://t.me/opdemy WEBSITE: www.opdemy.com

Railway and hauled by three steam locomotives – Sahib, Sindh and Sultan – ran for 34 kilometers with 400 people in 14 carriages on 1,676 mm broad gauge track between Bori Bunder (Mumbai) and Thane on 16 April 1853. The Thane viaducts, India’s first railway bridges were built over the Thane creek when the Mumbai-Thane line was extended to Kalyan in May 1854.

Q) With reference to “sessions of Parliament”, consider the following statements: 1. The decision to convene a session is taken by the Home Minister. 2. The summoning of Parliament is specified in Article 85 of the Constitution. Which of the statements given above is/are incorrect? A. 1 only B. 2 only C. Both 1 and 2 D. Neither 1 nor 2

ANS - A

The government has said that parties are in favor of doing away with the Winter Session, and that it would be appropriate to have the Budget Session in January.

Q) Consider the following statements: 1. The recently inaugurated Chilahati-Haldibari rail link would connect the borders of India & Myanmar. 2. It is being re-opened after a gap of 55 years.

Which of the statements given above is/are correct?

A. 1 only B. 2 only C. Both 1 and 2 D. Neither 1 nor 2

ANS - B

The recently inaugurated Chilahati-Haldibari rail link would connect the borders of India and Bangladesh. It is being reopened after 55 years after the rail links between India & then East Pakistan snapped during the 1965 war.

Q) Consider the following statements: 1. India has pledged a sum of USD 1 million to the World Anti-Doping Agency (WADA), towards its scientific research budget, to ensure an atmosphere of Clean Sport globally.

62 TELEGRAM LINK: https://t.me/opdemy WEBSITE: www.opdemy.com

2. The contribution made by India is the highest among contributions made by other world governments.

Which of the statements given above is/are correct?

A. 1 only B. 2 only C. Both 1 and 2 D. Neither 1 nor 2

ANS - C

Q) Which country has been banned from using its name, flag and anthem at the next two Olympics or any world championships for the next two years by the Court of Arbitration for Sport?

A. Iran B. Syria C. Russia D. North Korea

ANS - C

The Court of Arbitration for Sport's ruling has also blocked Russia from bidding to host any major sporting events for two years. Despite the ban on Russia, the Russian athletes and teams will be allowed to compete at all International events in the next two years as neutral athletes including the upcoming Tokyo Olympics, the 2022 Winter Games in Beijing, the world championships and the 2022 World Cup in Qatar, if they are not implicated in doping or covering up positive tests.

Q) The Indian Government held a virtual meeting "Bangabandhu-Bapu 2020" with which country to commemorate each other's Father of the Nation?

A. Bangladesh B. Maldives C. Sri Lanka D. Oman

ANS - A

Shri Narendra Modi, Prime Minister of the Republic of India and H.E. Sheikh Hasina, Prime Minister of the People’s Republic of Bangladesh held a Summit in virtual format on 17 December

63 TELEGRAM LINK: https://t.me/opdemy WEBSITE: www.opdemy.com

2020. Both sides held comprehensive discussions on all aspects of bilateral relations, and exchanged views on regional and international issues.

Q) Which State Government has launched ‘Pareshram’ Portal along with 22 online services of the Labour and Employees State Insurance Department that will be helpful in “Ease of Doing Business” and contribute significantly to the industrial development of the State?

A. Maharashtra Govt B. Odisha Govt C. Gujarat Govt D. Uttar Pradesh Govt

ANS - B

Q) Which central bank has launched FX 4 U that enables its entire internet banking users to handle forex transactions hassle-free?

A. State Bank of India B. Punjab National Bank C. Canara Bank D. Bank of India

ANS - C

FX 4 U is used for forex remittance through internet banking. With this development, eligible customers will be able to undertake remittance facility in adherence to the FEMA regulations 1999.

Q) Consider the following statements: 1. In 2008, the newly elected Constituent Assembly declared Nepal a Federal Democratic Republic, abolishing the 240-year-old monarchy. 2. Prime Minister K.P. Sharma Oli recommended the dissolution of Parliament of Nepal and called for the general election.

Which of the statements given above is/are correct?

A. 1 only B. 2 only C. Both 1 and 2 D. Neither 1 nor 2

ANS - C

64 TELEGRAM LINK: https://t.me/opdemy WEBSITE: www.opdemy.com

In an emergency Cabinet meeting held on Sunday morning, Prime Minister K.P. Sharma Oli recommended the dissolution of the Parliament of Nepal and called for the general election. The decision was ratified by President Bidhya Devi Bhandari in an official announcement hours later.

Q) Consider the following statements: 1. India is to bring more nations into the coastal radar network to include Maldives and Myanmar. 2. Mauritius, Seychelles, and Sri Lanka have already been integrated into the country’s coastal radar chain network

Which of the statements given above is/are correct?

A. 1 only B. 2 only C. Both 1 and 2 D. Neither 1 nor 2

ANS - B

As part of efforts to further expand the coastal radar chain network meant to enable real-time monitoring of the high seas for threats as also, expand India’s assistance for capacity building to the Indian Ocean littoral states, efforts are in advanced stages to set up coastal radar stations in the Maldives, Myanmar, and Bangladesh. Mauritius, Seychelles, and Sri Lanka have already been integrated into the country’s coastal radar chain network.

Q) Consider the following statements: 1. The first rescue and rehabilitation centre for monkeys in Telangana was inaugurated near Chincholi village in Nirmal district. 2. This is only the second such facility in the country after one presently available in Uttar Pradesh.

Which of the statements given above is/are correct?

A. 1 only B. 2 only C. Both 1 and 2 D. Neither 1 nor 2

ANS - A

65 TELEGRAM LINK: https://t.me/opdemy WEBSITE: www.opdemy.com

This is only the second such facility in the country after one presently available in Himachal Pradesh.

Q) Consider the following statements: 1. Nehru Zoological Park in Hyderabad is the first in India to get an ISO certification. 2. Arignar Anna Zoological Park is India's first and the largest Zoological Park.

Which of the statements given above is/are incorrect?

A. 1 only B. 2 only C. Both 1 and 2 D. Neither 1 nor 2

ANS - C

International Organization of Standardisation Certificate is given for quality management. Arignar Anna Zoological Park is India's first and the largest Zoological Park located in Chennai, Tamil Nadu.

Q) Consider the following statements: 1. France has recently become a member of the Indian Ocean Rim Association (IORA). 2. The headquarters of IORA is in New Delhi, India.

Which of the statements given above is/are incorrect?

A. 1 only B. 2 only C. Both 1 and 2 D. Neither 1 nor 2

ANS - B

The headquarters of IORA is at Port Louis, Mauritius.

Q) Consider the following statements: 1. India has bagged the 111th rank in the Human Freedom Index published by American think tank Cato Institute and Fraser Institute in Canada. 2. New Zealand topped the list and North Korea came last.

Which of the statements given above is/are correct?

66 TELEGRAM LINK: https://t.me/opdemy WEBSITE: www.opdemy.com

A. 1 only B. 2 only C. Both 1 and 2 D. Neither 1 nor 2

ANS - A

North Korea is not on the list. Syria bagged the last rank.

Q) ISRO's PSLV rocket launched the 42nd communication satellite CMS-01 of India into the orbit?

A. GSAT-12 B. GSAT-30 C. GSAT-31 D. GSAT-17

ANS - A

Q) The Asian Development Bank (ADB) has signed a $300 million loan agreement with the Government of India to upgrade rural power distribution networks in which Indian State?

A. Maharashtra B. Uttar Pradesh C. Tamil Nadu D. Karnataka

ANS - B

This is the first tranche of the loan for the Uttar Pradesh Power Distribution Network Rehabilitation Project.

Q) E20 fuel is a blend of 20% of ethanol with:

A. Gasoline B. Diesel C. Compressed natural gas D. Kerosene

ANS - A

The Ministry of Road Transport and

67 TELEGRAM LINK: https://t.me/opdemy WEBSITE: www.opdemy.com

Highways have published a draft notification dated 11th December 2020, seeking comments from the public for the adoption of E20 fuel as an automotive fuel and for the adoption of mass emission standards for this fuel. E20 fuel is a blend of 20% of ethanol with Gasoline. The notification facilitates the development of E20 compliant vehicles.

Q) Which government will launch a special program named "Kisaan Kalyan Mission" to double the income of farmers?

A. Maharashtra Govt B. Kerala Govt C. Uttar Pradesh D. Punjab Govt

ANS - C

Uttar Pradesh government will launch a special program to double the income of farmers. The program named as Kisan Kalyan Mission will cover all assembly constituencies of the state. Chief Minister Yogi Adityanath will today address a large farmers gathering in Bareilly to clear the misconception about the recent farm laws.

Q) Consider the following statements: 1. The government has decided to extend the suspension of the Insolvency and Bankruptcy Code (IBC) till March 31, 2021 2. The Code provides for a time-bound process to resolve insolvency.

Which of the statements given above is/are correct?

A. 1 only B. 2 only C. Both 1 and 2 D. Neither 1 nor 2

ANS - C

The government has decided to extend the suspension of the Insolvency and Bankruptcy Code (IBC) till March 31, 2021, to help businesses cope with the lingering difficulties posed by the COVID-19 pandemic, Union Finance Minister Nirmala Sitharaman said on Monday. The Minister’s statement settles a major uncertainty facing businesses since the present government order to keep the insolvency code in abeyance was due to expire on December 25.

68 TELEGRAM LINK: https://t.me/opdemy WEBSITE: www.opdemy.com

Q) Consider the following statements: 1. Mutation occurs due to changes in the genetic sequence of the virus, which in the case of SARS CoV 2, is the RNA; the mutant variant of the SARS CoV 2 is called N501Y. 2. The Indian Government has suspended all flights originating from the United Kingdom to India till December 31, 2020

Which of the statements given above is/are correct?

A. 1 only B. 2 only C. Both 1 and 2 D. Neither 1 nor 2

ANS - C

The recent infectious wave in the UK, prompting fresh lockdown measures, is associated with a mutant variant of the SARS CoV 2 called N501Y. Mutation occurs due to changes in the genetic sequence of the virus, which in the case of SARS CoV 2, is the RNA. This happens due to mistakes that occur when the virus is replicating. The mutation is capable of changing the course of the disease only if it results in a significant change in the protein structure.

Q) Consider the following statements: 1. The first successful and safe launch of the F/A-18 Super Hornet from a ski-jump begins the validation process to operate effectively from Indian Navy aircraft carriers 2. F/A-18 Super Hornet fighter jets are from France.

Which of the statements given above is/are correct?

A. 1 only B. 2 only C. Both 1 and 2 D. Neither 1 nor 2

ANS - A

F/A-18 Super Hornet fighter jets are from the United States. The F/A-18 Block III Super Hornet would not only provide superior war-fighting capability to the Indian Navy but also create opportunities for cooperation in naval aviation between the U.S. and India, Mr. Kanaglekar said, pitching it as a “lynchpin” for cooperation between Indian and U.S. Navies.

Q) Consider the following statements:

69 TELEGRAM LINK: https://t.me/opdemy WEBSITE: www.opdemy.com

1. India becomes the 4th country in the world to build a mega hypersonic wind tunnel after USA, Russia, and China. 2. The Project has been completed by DRDO in Hyderabad.

Which of the statements given above is/are incorrect?

A. 1 only B. 2 only C. Both 1 and 2 D. Neither 1 nor 2

ANS - A

India has become the 3rd country to have a mega hypersonic wind tunnel after the USA and Russia. China also has a hypersonic wind tunnel but its size is too small to be considered.

Q) Consider the following statements: 1. AMU became a University in 1920, through an Act of Indian Legislative Council by elevating the Mohammedan Anglo-Oriental (MAO) College to the status of a Central University. 2. MAO College was set up in 1910 by .

Which of the statements given above is/are correct?

A. 1 only B. 2 only C. Both 1 and 2 D. Neither 1 nor 2

ANS - A

Prime Minister Narendra Modi will address the centenary celebrations of Aligarh Muslim University on 22 December 2020. AMU became a University in 1920, through an Act of Indian Legislative Council by elevating the Mohammedan Anglo Oriental (MAO) College to the status of a Central University. MAO College was set up in 1877 by Sir .

Q) Consider the following statements: 1. The first Centre of Excellence (CoE) for ‘skill development in the power sector has been inaugurated in Gurugram. 2. The CoE has been set up on the campus of the National Institute of Solar Energy, Gurugram, Haryana.

70 TELEGRAM LINK: https://t.me/opdemy WEBSITE: www.opdemy.com

Which of the statements given above is/are correct?

A. 1 only B. 2 only C. Both 1 and 2 D. Neither 1 nor 2

ANS - C

Q) The Reserve Bank of India has further extended the restrictions on which crisis-ridden bank by three months till 31 March 2021?

A. Yes Bank B. PMC Bank C. UCO Bank D. HDFC Bank

ANS - B

The Administrator Of PMC Bank - AK Dixit PMC Bank Founded - 1984 PMC Bank Headquarters - Mumbai, Maharashtra

Q) Which newly discovered orchid has been named “the ugliest orchid in the world"?

A. Ancile Gloudon B. Gastrodia Agnicellus C. Bletilla Straitia D. Cypripedium calceolus

ANS - B

Found in a forest in Madagascar, it feeds on fungi. and has no leaves. Although assessed as a threatened species, the plants have some protection because they are located in a national park. Madagascar is an island country in the Indian Ocean, approximately 400 kilometers off the coast of East Africa. Madagascar is the world's second-largest island country.

Q) Consider the following statements: 1. Chhattisgarh Inclusive Rural and Accelerated Agriculture Growth (CHIRAAG), a $100 million projects have been approved by the World Bank. 2. The project aims at promoting nutrition-supportive agriculture for tribal households in Chhattisgarh.

71 TELEGRAM LINK: https://t.me/opdemy WEBSITE: www.opdemy.com

Which of the statements given above is/are correct?

A. 1 only B. 2 only C. Both 1 and 2 D. Neither 1 nor 2

ANS - C

Among the four approved projects is the $100 million Chhattisgarh Inclusive Rural and Accelerated Agriculture Growth Project (CHIRAAG) that will develop sustainable production systems that allow tribal households in remote areas of the state to practice round-the-year production of diversified and nutritious food.

Q) The Reserve Bank of India (RBI) has imposed a monetary penalty of Rs 50 lakh on which bank for non-compliance with directions issued on Income Recognition and Asset Classification (IRAC) norms?

A. PMC Bank B. Yes Bank C. UCO Bank D. HDFC Bank

ANS - C

RBI 25th Governor - Shaktikant Das Headquarters - Mumbai, Maharashtra Founded - 1 April 1935, Kolkata

Q) With reference to Great Indian Bustard (GIB), consider the following statements: 1. The Ministry of Environment Forest and Climate Change, along with the Wildlife Conservation Society, India, has launched a “firefly bird diverter” for GIB. 2. The GIB is one of the heaviest flying birds endemic to the Indian subcontinent.

Which of the statements given above is/are correct?

A. 1 only B. 2 only C. Both 1 and 2 D. Neither 1 nor 2

ANS - C

The Ministry of Environment Forest and Climate Change, along with the Wildlife Conservation Society, India, has come up with a unique

72 TELEGRAM LINK: https://t.me/opdemy WEBSITE: www.opdemy.com

initiative — a “firefly bird diverter” for overhead power lines in areas where Great Indian Bustard (GIB) populations are found in the wild. The GIB is one of the most critically threatened species in India, with fewer than 150 birds left in the wild.

Q) Consider the following statements: 1. Possession of a dead animal’s skin amount to an offense under Maharashtra Animal Preservation Act (MAPA), 1976 Bombay High Court said. 2. The MAPA prohibits slaughter, purchase, sale, import, export, and possession of Beef.

Which of the statements given above is/are correct?

A. 1 only B. 2 only C. Both 1 and 2 D. Neither 1 nor 2

ANS - B

Possession of a dead animal’s skin will not amount to an offense under the Maharashtra Animal Preservation Act (MAPA), 1976, the Nagpur Bench of the Bombay High Court has said while quashing an FIR against a man for possession of skins of dead cows.

Q) Consider the following statements: 1. The Western Ghats which includes 6 states Kerala, Karnataka, Goa, Maharashtra, Gujarat and Tamil Nadu - is home to 3,387 leopards 2. Gujarat tops the list, followed by Tamil Nadu according to the Status of Leopards in India 2018 report.

Which of the statements given above is/are correct?

A. 1 only B. 2 only C. Both 1 and 2 D. Neither 1 nor 2

ANS - A

The Western Ghats region is home to 3,387 leopards stealthily roaming around its forests. Karnataka tops the list with 1,783 leopards, followed by Tamil Nadu with 868, according to the Status of Leopards in India 2018 report. With 650 leopards, Kerala has the third-highest number of big cats in the Western Ghats region. Goa has 86.

73 TELEGRAM LINK: https://t.me/opdemy WEBSITE: www.opdemy.com

Q) With reference to the Winter Solstice, consider the following statements: 1. December 19 was Winter Solstice, the shortest day of the year in the Northern Hemisphere. 2. In the Southern Hemisphere December 21 was Summer Solstice, the year’s longest day.

Which of the statements given above is/are incorrect?

A. 1 only B. 2 only C. Both 1 and 2 D. Neither 1 nor 2

ANS - A

December 21 was Winter Solstice, the shortest day of the year in the Northern Hemisphere. In the Southern Hemisphere, conversely, December 21 was Summer Solstice, the year’s longest day.

Q) Consider the following statements: 1. PM Narendra Modi has been awarded "Legion of Merit" for drastically improving the Indo-US ties & emergence of India as a global superpower. 2. Australian Prime Minister Scott Morrison and Japanese Prime Minister Yoshihide Suga is also awarded the same.

Which of the statements given above is/are correct?

A. 1 only B. 2 only C. Both 1 and 2 D. Neither 1 nor 2

ANS - A

The award has been given to the former Prime Minister of Japan, Shinzo Abe, not to the incumbent Prime Minister Yoshihide Suga.

Q) Consider the following statements: 1. The 5th edition of the annual Global Technology Summit (GTS) was organized virtually by the Ministry of Science & Technology. 2. The theme of the 2020 GTS was “The Geopolitics of Technology”.

74 TELEGRAM LINK: https://t.me/opdemy WEBSITE: www.opdemy.com

Which of the statements given above is/are correct?

A. 1 only B. 2 only C. Both 1 and 2 D. Neither 1 nor 2

ANS - B

The 5th edition of the annual Global Technology Summit (GTS) was organized virtually by the Ministry of External Affairs (MEA).

Q) The Uttar Pradesh government has launched which special campaign to curb property & land-related disputes in rural areas?

A. Bhoomi Campaign B. Varasat Campaign C. Mitti Campaign D. Zameen Campaign

ANS - B

This is a first of its kind campaign in the state which aims to put an end the land-related issues in the rural areas and eradicate the exploitation of the villagers over succession rights by land mafias, who generally target disputed properties. The two-month-long special drive will continue till 15th February 2020.

Q) Which person has won the BBC Sports Personality of the Year 2020?

A. Virat Kohli B. Lewis Hamilton C. Jurgen Klopp D. Andrea Spendolini-Sirieix

ANS - B

Lewis Hamilton is the Formula 1 World Champion. This is the 2nd time he has won the award, the first time being in 2014.

Q) Consider the following statements: 1. The 20th edition of the Indian Ocean Rim Association (IORA) Council of Ministers (COM) Virtual Meeting was held under the Chairmanship of the United Arab Emirates (UAE). 2. The theme of the meeting was “Promoting a Shared Destiny and Path to

75 TELEGRAM LINK: https://t.me/opdemy WEBSITE: www.opdemy.com

Prosperity in the Indian Ocean”.

Which of the statements given above is/are correct?

A. 1 only B. 2 only C. Both 1 and 2 D. Neither 1 nor 2

ANS - C

The 20th edition of the Indian Ocean Rim Association (IORA) Council of Ministers (COM) Meeting via video conferencing under the Chairmanship of the United Arab Emirates (UAE). The theme of the meeting was “Promoting a Shared Destiny and Path to Prosperity in the Indian Ocean”. The Indian delegation was led by V. Muraleedharan, MoS, Ministry of External Affairs. Sri Lanka has assumed the charge of Vice-Chair of IORA for the period 2021-23.

Q) A ‘Joint Vision for Peace, Prosperity and People’ document was adopted and Hugh Speed Guard Boat Project along with $100 million Line of Credit was announced to guide the future development of Comprehensive Strategic Partnership between India and which ASEAN country?

A. Thailand B. Indonesia C. Philippines D. Vietnam

ANS - D

Prime Minister Narendra Modi held a Virtual Summit with H.E. Nguyen Xuan Phuc, Prime Minister of Vietnam. Both leaders also welcomed the signing of a Plan of Action for period 2021-2023 for further implementation of Comprehensive Strategic Partnership to implement the Joint Vision.

Q) Consider the following statements: 1. Union Cabinet has approved 100% Foreign Direct Investment (FDI) in direct-to-home (DTH) services 2. Earlier because of the guidelines issued by the Information and Broadcasting [I&B] Ministry, the FDI was limited to 75%.

Which of the statements given above is/are correct?

76 TELEGRAM LINK: https://t.me/opdemy WEBSITE: www.opdemy.com

A. 1 only B. 2 only C. Both 1 and 2 D. Neither 1 nor 2

ANS - A

Earlier because of the guidelines issued by the Information and Broadcasting [I&B] Ministry, the FDI was limited to 49%.

Q) Consider the following statements: 1. Jallikattu is an ancient sport belonging to Tamil Classical Age 2. Jallikattu is a sport conducted as a part of Mattu Pongal - 3rd day of the four-day-long festival Pongal.

Which of the statements given above is/are correct?

A. 1 only B. 2 only C. Both 1 and 2 D. Neither 1 nor 2

ANS - C

The Tamil Nadu government has permitted jallikattu to be held across the state during the coming Pongal season. But the bull-taming sport would be subject to fresh restrictions in view of the COVID-19 pandemic. According to an official press release, not more than 300 bull tamers would be allowed to take part in a jallikattu, ‘manjuvirattu’ or ‘vadamadu’ event. All the spectators would have to undergo thermal screening before entering the venue.

Q) Consider the following statements: 1. The Times of India won two golds and two silvers at WAN-IFRA’s South Asian Digital Media Awards. 2. The World Association of News Publishers, or WAN-IFRA, is the global organization of the world’s press. Which of the statements given above is/are correct? A. 1 only B. 2 only C. Both 1 and 2 D. Neither 1 nor 2

ANS - B

77 TELEGRAM LINK: https://t.me/opdemy WEBSITE: www.opdemy.com

The Hindu Group won two golds and two silvers at WAN-IFRA’s South Asian Digital Media Awards. The awards are presented in recognition of outstanding work done by news publishers in digital media. The Hindu #KeepTheHabit campaign won gold and silver in Best Native Advertising/ Branded Content Campaign and Best in Audience Engagement, respectively.

Q) Consider the following statements: 1. HelpAge India has been presented the UN Population Award for 2020 (institutional category), according to a release issued by UNFPA. 2. For the first time in the history of the UN Population Award, the honour is being conferred on an Indian institution.

Which of the statements given above is/are correct?

A. 1 only B. 2 only C. Both 1 and 2 D. Neither 1 nor 2

ANS - C

HelpAge India has been presented the UN Population Award for 2020 (institutional category), according to a release issued by UNFPA. For the first time in the history of the UN Population Award, the honour is being conferred on an Indian institution.

Q) Consider the following statements: 1. The Indian Government has inaugurated India's first Center of Excellence for skill development in the power sector in Gurugram. 2. The German Government also cooperated with this project.

Which of the statements given above is/are incorrect? A. 1 only B. 2 only C. Both 1 and 2 D. Neither 1 nor 2

ANS - B

The project has been completed with the cooperation of the French Government.

78 TELEGRAM LINK: https://t.me/opdemy WEBSITE: www.opdemy.com

UPSC CSE PRELIMS 2021

CRASH COURSE DATE SUBJECT 10/04/2021, 11/04/2021 Budget and Economic Survey

12/04/2021, 13/04/2021 Art and Culture

14/0/2021, 15/04/2021, Economics 16/04/2021

17/18/2021,18/04/2021, Modern Indian History 19/04/2021

20/04/2021,21/04/2021, Geography 22/04/2021

23/04/2021,24/04/2021, Enviroment 25/04/2021

26/04/2021,27/04/2021, 28/04/2021,29/04/2021, Polity 30/04/2021

01/05/2021, 02/05/2021 Science and Tech *Note: Also, there will be Lectures on CSAT as and when time permits* PRICE: 1,999 For Any Queries: 7988797680, 7999136838, 6295118940

79 TELEGRAM LINK: https://t.me/opdemy WEBSITE: www.opdemy.com

Q) Consider the following statements: 1. The 12th GRIHA (Green Rating for Integrated Habitat Assessment) Summit was virtually inaugurated by the Vice President of India Venkaiah Naidu. 2. The theme of the Summit was “Rejuvenating Resilient Habitats”.

Which of the statements given above is/are correct?

A. 1 only B. 2 only C. Both 1 and 2 D. Neither 1 nor 2

ANS - C

Q) Which bank has announced to provide Rs 2,100 crore loan to the Tripura government for the development of urban areas and tourism in the northeastern state?

A. World Bank B. Asian Development Bank C. International Monetary Fund D. New Development Bank

ANS - B

From the total fund of Rs 2,100 crore, Rs 1600 crore will be for urban development and Rs 500 crore for the tourism sector.

ABOUT ADB Headquarters - Mandaluyong, Philippines President - Masatsugu Asakawa Founded - 19 December 1966 Purpose - Economic development Main organ - Board of Governors

Q) Prime Minister Narendra Modi will launch ‘SEHAT’ – Health Insurance Scheme on 26th December for which Union Territory?

A. New Delhi B. Chandigarh

80 TELEGRAM LINK: https://t.me/opdemy WEBSITE: www.opdemy.com

C. Ladakh D. Jammu & Kashmir

ANS - D

This scheme will cover the remaining One crore population which was not covered under Ayushman Bharat Scheme. Under Ayushman Bharat PM Jan Arogya Yojana (PMJAY), eligible beneficiaries were covered under Health Cover of Rs 5 Lakh. Under the PMJAY scheme, 30 lakh people are being covered in J&K. With the launch of the ‘SEHAT’ Scheme by the Prime Minister on December 26, J&K will be among the first in the country to achieve Universal Health Coverage. J&K Lieutenant Governor - Manoj Sinha

Q) Consider the following statements: 1. The President of India held a virtual 16th convocation at King George's Medical University located in New Delhi. 2. The University was in the news for helping in India's fight against COVID-19.

Which of the statements given above is/are correct?

A. 1 only B. 2 only C. Both 1 and 2 D. Neither 1 nor 2

ANS - B

The university is located in Lucknow, Uttar Pradesh, and was set up in 1905.

Q) Where did the ONGC launch India's 8th hydrocarbon producing basin?

A. Kerala B. West Bengal C. Maharashtra D. Karnataka

ANS - B

ONGC begins production in the Bengal basin, making it India's eighth functional. Oil and Natural Gas Corporation (ONGC) Limited has begun crude oil

81 TELEGRAM LINK: https://t.me/opdemy WEBSITE: www.opdemy.com

production from the

Q) With reference to high-security registration plates (HSRP), consider the following statements: 1. Ministry of Road Transport and Highways (MoRTH) has mandated all vehicles sold in India before April 2019 to have high-security registration plate (HSRP) 2. These are made up of chromium and contains 8-digit permanent identification number (PIN)

Which of the statements given above is/are correct?

A. 1 only B. 2 only C. Both 1 and 2 D. Neither 1 nor 2

ANS - A

High-security registration plates are made up of aluminum and are affixed using at least two non-reusable snap-on locks. The plate comprises a hot-stamped chromium-based 20 mm X 20 mm hologram of Ashoka Chakra in blue color on the top left corner. At the bottom left corner is a 10-digit permanent identification number (PIN) that is laser branded on the reflective sheeting. HSRP also includes a hot stamped film applied on the numerals and letters bearing the inscription "INDIA" at a 45-degree angle. Once affixed, the unique high-security registration plate is electronically linked to the vehicle.

Q) Consider the following statements: 1. U.K is only the second country to leave the E.U. after Switzerland. 2. Switzerland is not a member of the European Union. However, it has signed a number of treaties with the European Union.

Which of the statements given above is/are correct?

A. 1 only B. 2 only C. Both 1 and 2 D. Neither 1 nor 2

ANS - B

On January 31, 2020, the United Kingdom (U.K) formally left the European Union. U.K is the first country to leave the E.U. The exit was in accordance with Article 50 of the Treaty of the European Union.

82 TELEGRAM LINK: https://t.me/opdemy WEBSITE: www.opdemy.com

Q) Consider the following statements: 1. M. G. Ramachandran was an Indian politician and film actor who served as the Chief Minister of Tamil Nadu for ten years between 1977 and 1987 2. He was awarded India's highest civilian honor, the Bharat Ratna.

Which of the statements given above is/are correct?

A. 1 only B. 2 only C. Both 1 and 2 D. Neither 1 nor 2

ANS - C

M.G. Ramachandran, or MGR (1917-1987), founder of Tamil Nadu’s ruling party, the All India Anna Dravida Munnetra Kazhagam (AIADMK), and Chief Minister for an almost unbroken period of 10 years (1977-87)

Q) Consider the following statements: 1. National Farmers’ Day, or Kisan Diwas, is celebrated across the country on December 23 2. It is to honor India’s farmers and mark the birth anniversary of the nation’s 1st Agriculture minister of India -

Which of the statements given above is/are correct?

A. 1 only B. 2 only C. Both 1 and 2 D. Neither 1 nor 2

ANS - A

It is to honor India’s farmers and mark the birth anniversary of the nation’s fifth prime minister, Chaudhary Charan Singh.

Q) Consider the following statements: 1. Tata Motors has launched a "Go Green" initiative. 2. The initiative will consist of minimizing and then ultimately ending the use of harmful materials used for the production of four-wheeled vehicles.

Which of the statements given above is/are incorrect?

A. 1 only

83 TELEGRAM LINK: https://t.me/opdemy WEBSITE: www.opdemy.com

B. 2 only C. Both 1 and 2 D. Neither 1 nor 2

ANS - A

According to the initiative, the company will plant one tree for every car sold.

Q) Consider the following statements: 1. Indigenously developed Brazilian Remote Sensing Satellite, Amazonia-I is scheduled to be launched on a PSLV (Polar Satellite ) of ISRO early next year. 2. This has been designed and assembled locally in Brazil. Tested in Brazil this will be the first satellite for Earth Observation.

Which of the statements given above is/are correct?

A. 1 only B. 2 only C. Both 1 and 2 D. Neither 1 nor 2

ANS - C

Q) Which college/university has been recognized with the ‘Award of Merit’ in the 2020 UNESCO Asia-Pacific Awards for Cultural Heritage Conservation?

A. Jawaharlal Nehru University B. King George's Medical College C. Amar Singh College D. Banaras Hindu University

ANS - C

Srinagar based Amar Singh College has been recognized with the ‘Award of Merit’ in the 2020 UNESCO Asia-Pacific Awards for Cultural Heritage Conservation. The restoration of the Amar Singh College brought back one of the most prominent institutional buildings in Kashmir to its former glory.

Q) Which international bank has signed a USD 500 million project to build safe and green national highway corridors in Rajasthan, Himachal Pradesh, Uttar Pradesh, and Andhra Pradesh?

84 TELEGRAM LINK: https://t.me/opdemy WEBSITE: www.opdemy.com

A. Asian Development Bank B. World Bank C. International Monetary Fund D. New Development Bank

ANS - B

The Green National Highways Corridors project will support the Road Transport and Highways Ministry to construct 783 kilometers of highways in various geographies by integrating safe and green technology designs such as local and marginal materials, industrial by-products and other bioengineering solutions. The project will help reduce GHG emissions in the construction and maintenance of highways.

Q) Consider the following statements: 1. National Consumer Rights Day is observed on 24th December every year. 2. The theme of this year's National Consumer Rights Day is "Sustainable Consumer". Which of the statements given above is/are correct? A. 1 only B. 2 only C. Both 1 and 2 D. Neither 1 nor 2

ANS - C

World's Consumer Rights Day is observed on 15th March every year.

Q) Which bank has signed a memorandum of understanding (MoU) with the Indian Navy and the Indian Coast Guard under which the bank would offer customized services along with a host of facilities to account holders?

A. State Bank of India B. Bank of Baroda C. Bank of India D. HDFC Bank

ANS - B

The bank has also renewed its existing MoU with the Indian Army to offer specially customized banking services through ”Baroda Military Salary Package”. Bank Of Baroda Headquarters: Vadodara, Gujarat, India.

85 TELEGRAM LINK: https://t.me/opdemy WEBSITE: www.opdemy.com

Bank Of Baroda Chairman: Hasmukh Adhia. Bank Of Baroda MD & CEO: Sanjiv Chadha.

Q) Consider the following statements. 1. Maitri and Himadri are two Indian permanent stations in Antarctica. 2. National Centre for Polar and Ocean Research (NCPOR) is the nodal agency for India’s scientific expeditions in Antarctica and the Arctic.

Which of the above statements is/are correct?

A. 1 only B. 2 only C. Both 1 and 2 D. Neither 1 nor 2

ANS - B

The two Indian permanent stations, Maitri and Bharati are in Antarctica. National Centre for Polar and Ocean Research (NCPOR) is the nodal agency for India’s scientific expeditions in Antarctica and the Arctic.

Q) Consider the following statements regarding Leadership Group on Industry Transition. 1. The Leadership Group for Industry Transition (LeadIT) gathers countries and companies that are committed to reaching net-zero carbon emissions from industry by 2050. 2. India is a part of this Grouping. 3. It was launched with support from the World Economic Forum (WEF).

Which of the above statements is/are correct?

A. 1, 2 B. 1, 3 C. 1, 2, 3 D. 2, 3

ANS - C

The Leadership Group for Industry Transition (LeadIT) gathers countries and companies that are committed to reaching net-zero carbon emissions from industry by 2050. The governments of Sweden and India, with support from the World Economic Forum (WEF) launched the Leadership Group for Industry Transition (LeadIT) at the UN Secretary General’s Climate Action Summit in September 2019. LeadIT is

86 TELEGRAM LINK: https://t.me/opdemy WEBSITE: www.opdemy.com

grounded in the conviction that partnership between the public and private sectors are key to achieving the industrial transition.

Q) Consider the following statements: 1. PM Kisan Samman Nidhi is an initiative by the government of India in which all farmers will get up to ₹6,000 per year as minimum income support. 2. The initiative was to be announced by Nirmala Sitharaman during 2019 Interim Union Budget of India on 1 February 2019.

Which of the statements given above is/are correct? A. 1 only B. 2 only C. Both 1 and 2 D. Neither 1 nor 2

ANS - A

PM Kisan Samman Nidhi Scheme was announced by Piyush Goyal during the Interim Union Budget of India on 1 February 2019. The scheme aims to provide income support to farmers for easing their liquidity needs to facilitate timely access to inputs.

Q) Which State Government unveiled the Farmer Registration and Unified beneficiary Information System (FRUITS), an e-governance portal, to create a repository of farmland information and farm loan details on a single platform?

A. Haryana Government B. Punjab Government C. Karnataka Government D. Maharashtra Government

ANS - C

All farmers will be registered and given an identification number on the portal. The FRUITS Portal is integrated into the BHOOMI Package of the Karnataka State for fetching and validating the land details. One other important facility of this Portal is online creation of charge by which visits of farmers to the Sub-Registrar Office are avoided. Canara Bank has agreed to run FRUITS on a pilot basis.

Q) With reference to the ammonia, consider the following statements: 1. A tri-hydroid of nitrogen (NH3), ammonia is a building block for ammonium nitrate (NH4NO3) that is used in agriculture as a high-nitrogen fertilizer. 2. Ammonia is stored for industrial use in liquid form under high pressure or in gaseous form at low temperature.

87 TELEGRAM LINK: https://t.me/opdemy WEBSITE: www.opdemy.com

Which of the statements given above is/are correct?

A. 1 only B. 2 only C. Both 1 and 2 D. Neither 1 nor 2

ANS - C

Two persons died and several took ill in a major ammonia gas leakage at the Indian Farmers Fertilizer Cooperative Limited (IFFCO) unit at Prayagraj. A tri-hydroid of nitrogen (NH3), ammonia is a building block for ammonium nitrate (NH4NO3) that is used in agriculture as a high-nitrogen fertilizer. Ammonia is stored for industrial use in liquid form under high pressure or in gaseous form at low temperature. The gas is toxic and affects the lungs with a possibility of causing chemical pneumonitis — inflammation of the lung caused by aspirating or inhaling irritants.

Q) The Union Cabinet approved revised guidelines for 100 percent foreign direct investment (FDI) as well as increasing the license period to 20 years for which services?

A. Hotel Services B. Telecom Services C. Railways D. DTH Services

ANS - D

Minister of Information and Broadcasting: Prakash Javadekar

Q) The Army version of the MRSAM Surface to Air missile is jointly being developed by India and which other country?

A. France B. Russia C. Israel D. Japan

ANS - C

The Defence Research & Development Organisation (DRDO) of India and Israel Aerospace Industries (IAI) of Israel have jointly developed the missile. It was recently tested on the coast of Odisha.

Q) Which of these statements is/are correct?

88 TELEGRAM LINK: https://t.me/opdemy WEBSITE: www.opdemy.com

1) "Swachta Abhiyan" is a mobile app recently launched by the Ministry of Social Justice and Empowerment. 2) Its purpose is to locate and identify the unsanitary latrine and manual scavengers do as make them into sanitary ones.

A. Only 1 B. Only 2 C. Both D. None

ANS - C

Q) Which of these statements are correct? 1) Recently, the Cabinet Committee on Economic Affairs has approved major changes in which of the following Centrally Sponsored Scheme of Post-Matric Scholarship Scheme for SC Students. 2) Around Rs. 59,000 crore has been released under this scheme for the education of SC Students.

A. 1 only B. 2 only C. Both 1 and 2 D. Neither 1 nor 2

ANS - C

Q) Which of these statements are correct? a) India's First Digital Asset Management Platform is Digibox. b) It was launched by the CEO of NITI Aayog.

A. Only 1 B. Only 2 C. Both D. None

ANS - C

Q) With reference to Cotton Corporation of India, consider the following statements. 1. The Cotton Corporation of India Limited or CCI is a Government of India agency, engaged in diverse activities related to trade, procurement, and export of cotton. 2. CCI is governed by Textile Policy 1985 issued by the Ministry of Textiles, Government of India.

Which of the above statements is/are correct?

89 TELEGRAM LINK: https://t.me/opdemy WEBSITE: www.opdemy.com

A. 1 only B. 2 only C. Both 1 and 2 D. Neither 1 nor 2

ANS - C

Former Union Minister and Shiromani Akali Dal (SAD) leader Harsimrat Kaur Badal on Sunday said the apprehensions of farmers were already coming true in Punjab, with the Cotton Corporation of India (CCI) placing daily ceilings on the procurement of cotton and effecting a four-times reduction in its purchase.

Q) Consider the following statements. 1. Eminent dance scholar and critic Sunil Kothari passed away at the age of 87. 2. He was a Padma Shri winner who wrote extensively on Bharatanatyam, Kathak, and Manipuri dance forms.

Which of the above statements is/are correct?

A. 1 only B. 2 only C. Both 1 and 2 D. Neither 1 nor 2

ANS - C

Eminent dance scholar and critic Sunil Kothari, 87, passed away on Sunday morning in a private hospital in Delhi because of COVID-19-related complications. A Sangeet Natak Akademi fellow, Kothari blazed a new trail in documenting Indian classical dance forms. Born in the Kheda district of Gujarat, Kothari qualified as a chartered accountant and taught in Mumbai’s Sydenham College for a brief while before turning to dance studies.

Q) With reference to Inner-Line Permit, consider the following statements: 1. Under ILP, five Northeastern states are covered, namely, Arunachal Pradesh, Mizoram, Manipur, Sikkim, and Nagaland. 2. An ILP is only valid for domestic tourists.

Which of the statements given above is/are correct?

A. 1 only B. 2 only C. Both 1 and 2

90 TELEGRAM LINK: https://t.me/opdemy WEBSITE: www.opdemy.com

D. Neither 1 nor 2

ANS - B

Implemented under the British-era Bengal Eastern Frontier Regulation, the ILP is an official document issued to let an Indian citizen enter a protected area for a limited period. Pressure groups in the northeast view this permit as a shield against the entry of illegal immigrants. At present, four Northeastern states are covered, namely, Arunachal Pradesh, Mizoram, Manipur and Nagaland.

Q) With reference to an annual report of Centre for Economics and Business Research on the global economy, consider the following statements: 1. Russia will overtake the United States to become the world's biggest economy in 2028. 2. The United Kingdom, currently the fifth-biggest economy by the CEBR's measure, would slip to sixth place from 2024.

Which of the statements given above is/are correct?

A. 1 only B. 2 only C. Both 1 and 2 D. Neither 1 nor 2

ANS - B

China will overtake the United States to become the world's biggest economy in 2028, the Centre for Economics and Business Research, a UK-based think tank said in an annual report. Japan would remain the world's third-biggest economy, in dollar terms, until the early 2030s, when it would be overtaken by India, pushing Germany down from fourth to fifth. The United Kingdom, currently the fifth-biggest economy by the CEBR's measure would slip to sixth place from 2024.

Q) Consider the following statements: 1. India’s biggest hockey stadium is to be set up in Rourkela, Odisha. 2. Odisha will be hosting FIH Men’s Hockey World Cup for a second consecutive time, in 2023, at two venues, Bhubaneswar and Rourkela.

Which of the statements given above is/are correct?

A. 1 only B. 2 only C. Both 1 and 2 D. Neither 1 nor 2

ANS - C

91 TELEGRAM LINK: https://t.me/opdemy WEBSITE: www.opdemy.com

Odisha Chief Minister Naveen Patnaik on Thursday announced that India's biggest hockey stadium will come up at Rourkela and the 20,000-capacity facility will host matches of FIH men's World Cup in 2023. Spread over 15 acres of land, the stadium will be constructed on the Biju Patnaik University of Technology campus.

Q) Himachal Pradesh Chief Minister Jai Ram Thakur has unveiled an 18-feet tall statue of which late former Prime Minister on the occasion of his/her 96th birth anniversary at the historic Ridge in the state capital?

A. Jawaharlal Nehru B. Indira Gandhi C. Lal Bahadur Shashtri D. Atal Bihari Vajpayee

ANS - D

Chief Minister of HP - Jairam Thakur Governor of HP - Bandaru Dattatreya

Q) The Union Minister for Skill Development and Entrepreneurship, Mahendra Nath Pandey has inaugurated the first batch of Indian Institute of Skill in which city?

A. Mumbai B. Kolkata C. Chennai D. Hyderabad

ANS - A

Q) India's first Tiger Reserve hot air balloon safari has been launched in which of the following Tiger Reserves?

A. Jim Corbett Tiger Reserve B. Ranthambore Tiger Reserve C. Sundarban Tiger Reserve D. Bandhavgarh Tiger Reserve

ANS - D

Tiger Reserves and their states Jim Corbett Tiger Reserve - Uttarakhand Ranthambore Tiger Reserve - Rajasthan

92 TELEGRAM LINK: https://t.me/opdemy WEBSITE: www.opdemy.com

Sundarban Tiger Reserve - West Bengal Bandhavgarh Tiger Reserve - Madhya Pradesh

Q) Which of these statements are correct? 1) India's first Lithium refinery is soon going to be set up in Uttar Pradesh. 2) Australia is the largest producer of Lithium & controls more than half the global supply.

A. 1 only B. 2 only C. Both 1 and 2 D. Neither 1 nor 2

ANS - B

Lithium is a rare material that is not usually found in India. India's first Lithium refinery is to be set up in Gujarat. Australia produces and controls around 53% of the total Lithium.

Q) Which is the 6th state to successfully undertake the "Ease of Doing Business" reforms as stipulated by the Ministry of Finance?

A. Uttar Pradesh B. Rajasthan C. Madhya Pradesh D. Punjab

ANS - B

Q) Consider the following statements. 1. Senior-most officer martyred in the Bangladesh Liberation War 1971 Brigadier Mohammad Usman was called Naushera ka Sher. 2. He was posthumously awarded the Maha Vir Chakra for his gallant action. Which of the above statements is/are correct?

A. 1 only B. 2 only C. Both 1 and 2 D. Neither 1 nor 2

ANS - B

Naushera ka Sher Brigadier Mohammad Usman, the senior-most officer was martyred in the first India-Pakistan war of 1947-48 Brig. Usman was one of only 18 Brigadiers in the Army at the

93 TELEGRAM LINK: https://t.me/opdemy WEBSITE: www.opdemy.com

time of Independence and was posthumously awarded the Maha Vir Chakra for his gallant action during the 1947-48 war.

Q) Consider the following statements. 1. Efforts are being made for instant demolition technique in place of extended demolition to avoid air pollution during the demolition process in Central Vista Project. 2. A nod from expert panels is necessary for projects to get green clearances from the central government's Ministry of Environment, Forest and Climate Change (MoEFCC).

Which of the above statements is/are correct?

A. 1 only B. 2 only C. Both 1 and 2 D. Neither 1 nor 2

ANS - C

An expert committee of the Union Ministry of Environment and Forest has asked the Central Public Works Department (CPWD) to submit detailed plans for the demolition of government buildings for its proposed redevelopment of the Central Vista, with a focus on “instant demolition” techniques to reduce pollution. A nod from the expert panels – the National Board for Wildlife (NBWL), the Forest Advisory Committee (FAC) and 10 Expert Appraisal Committees (EAC) – is necessary for projects to get green clearances from the central government's Ministry of Environment, Forest and Climate Change (MoEFCC).

Q) With reference to District Development Councils (DDCs) in J&K, consider the following statements: 1. The DDCs replace the District Planning and Development Boards (DDBs) 2. The term of the DDC will be five years but the Electoral process will not allow for reservations for Scheduled Castes, Scheduled Tribes, and women.

Which of the statements given above is/are incorrect?

A. 1 only B. 2 only C. Both 1 and 2 D. Neither 1 nor 2

ANS - B

District Development Councils in J&K will have elected representatives from each district. The term of the DDC will be five years Electoral process will allow for reservations for Scheduled Castes, Scheduled Tribes and women.

94 TELEGRAM LINK: https://t.me/opdemy WEBSITE: www.opdemy.com

Q) With reference to Eastern Dedicated Freight Corridor, consider the following statements: 1. The Eastern Dedicated Freight Corridor does not pass through Chhattisgarh. 2. The most distance covered in this freight corridor is in Uttar Pradesh.

Which of the statements given above is/are correct?

A. 1 only B. 2 only C. Both 1 and 2 D. Neither 1 nor 2

ANS - C

The states that include this project is Punjab, Haryana, Uttar Pradesh, West Bengal, Bihar & Jharkhand.

Q) Consider the following statements: 1. India's first-ever driverless rail network will be flagged off in the Magenta line of Delhi Metro. 2. The world's first-ever automated driverless rail network was set up in the US.

Which of the statements given above is/are incorrect?

A. 1 only B. 2 only C. Both 1 and 2 D. Neither 1 nor 2

ANS - B

The World's first-ever automated driverless rail network was set up in Japan called Port Island Line.

Q) Which State Cabinet has approved the "Dharm Swatantrya (Religious Freedom) Bill, 2020" to curb forceful religious conversions?

A. Uttar Pradesh B. Madhya Pradesh C. Gujarat D. Bihar

ANS - B

95 TELEGRAM LINK: https://t.me/opdemy WEBSITE: www.opdemy.com

Q) With which country has India signed an agreement to focus on environmental friendly medical practices called "Green Healthcare"?

A. Japan B. United Kingdom C. The US D. Israel

ANS - D

Q) Which of the following agency has launched a "Jhatpat Processing"?

A. Income Tax Department B. Delhi Police C. Supreme Court D. Indian Railways

ANS - A

To fast track the process of filing income tax, the Income Tax department has launched this unique initiative. The slogan of this initiative is "File Karo Jhat se, Processing Hogi Pat se".

Q) Which of these statements are correct? 1) The Indian Navy on Sunday concluded a two-day passage exercise with the Indonesian Navy in the South China Sea. 2) The capital of Indonesia is Jakarta and the currency is the Indonesian Rupiah.

A. 1 only B. 2 only C. Both 1 and 2 D. Neither 1 nor 2

ANS - A

The two-day passage exercise was conducted between the Indian and Vietnamese Navy in the South China Sea.

Q) Union Home Minister Amit Shah launched Thoubal Multipurpose Project in

A. Manipur B. Sikkim

96 TELEGRAM LINK: https://t.me/opdemy WEBSITE: www.opdemy.com

C. Assam D. West Bengal

ANS - A

Union Home Minister Amit Shah launched several development projects in Manipur. He inaugurated the e-office and Thoubal Multipurpose Project (Thoubal Dam) in Imphal through virtual mode which will irrigate 35,104 hectares. He also laid the foundation stone of projects, including the Churachandpur Medical College, IT-SEZ at Mantripukhri, Manipur Bhawan in New Delhi and the Integrated Command and Control Center at Imphal.

Q) With reference to Border Security Force (BSF), consider the following statements. 1. BSF is India's Primary border guarding organization on its border with Pakistan, Bangladesh, and Nepal. 2. BSF has been termed as the First Line of Defence of Indian Territories Which of the above statements is/are correct?

A. 1 only B. 2 only C. Both 1 and 2 D. Neither 1 nor 2

ANS - B

BSF is India's Primary border guarding organization on its border with Pakistan and Bangladesh.

Q) Consider the following statements. 1. India is planning to increase its renewable energy capacity to 450 GW by 2030. 2. India’s renewable energy capacity was the fourth largest in the world.

Which of the above statements is/are correct?

A. 1 only B. 2 only C. Both 1 and 2 D. Neither 1 nor 2

ANS - C

The Prime Minister mentioned India's achievement of meeting the goal of 175 gigawatts of renewable energy before the target of 2022, and promised "a big step ahead by seeking to achieve 450 gigawatts by 2030. India had reduced its emissions intensity by 21% since 2005. Installed solar capacity has grown to 36 gigawatts in 2020. The country’s renewable energy capacity was the fourth largest in the world and would reach 175GW before 2022.

97 TELEGRAM LINK: https://t.me/opdemy WEBSITE: www.opdemy.com

UPSC CSE PRELIMS 2021

CRASH COURSE DATE SUBJECT 10/04/2021, 11/04/2021 Budget and Economic Survey

12/04/2021, 13/04/2021 Art and Culture

14/0/2021, 15/04/2021, Economics 16/04/2021

17/18/2021,18/04/2021, Modern Indian History 19/04/2021

20/04/2021,21/04/2021, Geography 22/04/2021

23/04/2021,24/04/2021, Enviroment 25/04/2021

26/04/2021,27/04/2021, 28/04/2021,29/04/2021, Polity 30/04/2021

01/05/2021, 02/05/2021 Science and Tech *Note: Also, there will be Lectures on CSAT as and when time permits* PRICE: 1,999 For Any Queries: 7988797680, 7999136838, 6295118940

98 TELEGRAM LINK: https://t.me/opdemy WEBSITE: www.opdemy.com

Q) Consider the following statements: 1. ‘Digital Waters’ platform was launched by Indian National Centre for Oceanic Information Services (INCOIS) 2. It will play a central role in sustainable management of our oceans and expanding our ‘Blue Economy’ initiatives

Which of the statements given above is/are incorrect?

A. 1 only B. 2 only C. Both 1 and 2 D. Neither 1 nor 2

ANS - A

Union Minister for Science and Technology Harsh Vardhan launched the ‘Digital Ocean’ platform of the Indian National Centre for Oceanic Information Services (INCOIS) here as a one-stop-solution for all data related needs of a wide range of users, including research institutions, operational agencies, strategic users, academic community, maritime industry, and the public.

Q) Consider the following statements: 1. Japan has adopted a green growth plan to go carbon-free by the year 2050. 2. The currency of Japan is Yen.

Which of the statements given above is/are correct?

A. 1 only B. 2 only C. Both 1 and 2 D. Neither 1 nor 2

ANS - C

Japan aims to eliminate gasoline-powered vehicles in about 15 years, the government said Friday in a plan to achieve Prime Minister Yoshihide Suga's ambitious pledge to go carbon-free by 2050 and generate nearly USD 2 trillion growth in green business and investment.

Q) Bhashan Char island recently seen in the news is which country ??

99 TELEGRAM LINK: https://t.me/opdemy WEBSITE: www.opdemy.com

A. Myanmar B. Bangladesh C. Sri Lanka D. Thailand

ANS - B

Seven Bangladesh Navy ships carrying 1,804 Rohingya refugees arrived on Tuesday at an isolated island where they will be relocated despite concerns among human rights groups about their safety. They reached Bhashan Char island, 34 km from the mainland, after a four-hour naval journey from the port city of , government official Mohammed Khurshed Alam Khan said.

Q) The Union Health Minister Harsh Vardhan inaugurated India's first pneumococcal a conjugate vaccine called -

A. Pneumoccine B. Pneumosil C. Pneumonaccine D. Conjugate Pneumola

ANS - B

Q) Army chief General Manoj Mukund Naravane is on a three-day visit to which country to bolster military cooperation?

A. Saudi Arabia B. Myanmar C. South Korea D. United Arab Emirates

ANS - C

Q) Which is the first private company of India to successfully design, develop and test a full solid-fueled rocket stage?

A. Airnetz Aviation Pvt Ltd. B. Aroon Aviation Services Pvt Ltd. C. Hindustan Aeronautics Limited D. Skyroot Aerospace

ANS - D

Indian Space startup, Skyroot

100 TELEGRAM LINK: https://t.me/opdemy WEBSITE: www.opdemy.com

Aerospace has successfully test-fired a solid propulsion rocket engine, named Kalam-5. With this, it has become the first private company of India to successfully design, develop and test a full solid- fueled rocket stage. This crucial propulsion technology will be used for their maiden rocket Vikram-1, which is underactive manufacturing and is targeted to be launched in December 2021 with the help of ISRO.

Q) Which of these statements are correct? 1) MS Dhoni has been declared the "ICC Male Cricketer of the Decade". 2) Ellyse Perry has been awarded the "ICC Female of the Decade" award.

A. 1 only B. 2 only C. Both 1 and 2 D. Neither 1 nor 2

ANS - B

Virat Kohli has been declared the "ICC Male Cricketer of the Decade".

Q) Which of these statements are correct? 1) "Kisan Fasal Rahat Yojana" is a new insurance scheme launched by the West Bengal Govt. 2) The scheme will replace the existing "PM Kisaan Bima Yojana" in the state.

A. 1 only B. 2 only C. Both 1 and 2 D. Neither 1 nor 2

ANS - B

The scheme is being implemented by the Jharkhand Govt.

Q) Consider the following statements. 1. After three years, a tableau depicting a redeveloped Chandni Chowk — is likely to roll down Rajpath during the Republic Day celebrations. 2. Chandni Chowk (Moonlight Square) is one of the oldest and busiest markets in Old Delhi built in the 17th century by the Mughal Emperor of India Shah Jahan.

Which of the above statements is/are correct?

A. 1 only B. 2 only C. Both 1 and 2

101 TELEGRAM LINK: https://t.me/opdemy WEBSITE: www.opdemy.com

D. Neither 1 nor 2

ANS - C

The Chandni Chowk (Moonlight Square) is one of the oldest and busiest markets in Old Delhi, India. It was built in the 17th century by Mughal Emperor of India Shah Jahan and designed by his daughter Jahanara. After three years, a tableau proposed by the Delhi government — depicting a redeveloped Chandni Chowk — is likely to roll down Rajpath during the Republic Day celebrations.

Q) Consider the following statements. 1. Union Cabinet has approved the export of the indigenously developed and manufactured Akash short-range Surface to Air Missile (SAM) system. 2. Akash is developed by Hindustan Aeronautics Limited.

Which of the above statements is/are correct?

A. 1 only B. 2 only C. Both 1 and 2 D. Neither 1 nor 2

ANS - A

Akash is developed by the Defence Research and Development Organization

Q) Consider the following statements: 1. The first indigenous vaccine for pneumococcal conjugate was launched by Union Health Minister Dr. Harsh Vardhan. 2. The vaccine was developed by Bharat Biotech International Ltd.

Which of the statements given above is/are incorrect?

A. 1 only B. 2 only C. Both 1 and 2 D. Neither 1 nor 2

ANS - B

The vaccine was developed by the Serum Institute of India.

Q) Consider the following statements: 1. Inner Line Permit (ILP) is an official travel document issued by the concerned state

102 TELEGRAM LINK: https://t.me/opdemy WEBSITE: www.opdemy.com

government to allow inward travel of an Indian citizen into a protected area for a limited period. 2. Sikkim doesn’t come under the ILP regime. Which of the statements given above is/are correct?

A. 1 only B. 2 only C. Both 1 and 2 D. Neither 1 nor 2

ANS - C

Q) With reference to the Kisan Rail, consider the following statements: 1. The loading and unloading of perishable commodities will be permitted at all en route stoppages and no minimum or maximum size of the consignment. 2. The government has extended a subsidy of 50 percent on transportation of fruits and vegetables.

Which of the statements given above is/are correct?

A. 1 only B. 2 only C. Both 1 and 2 D. Neither 1 nor 2

ANS - C

Prime Minister Narendra Modi flagged off the 100th Kisan Rail train, which is also plying between Devlali and Danapur — a popular offering which has now been extended to run between Sangola to Muzaffarpur and has carried over 15,450 tonnes of produce. The Kisan Rail train carries vegetables such as cauliflower, capsicum, cabbage, drumsticks, chilies, onion, etc, as well as fruits like grapes, oranges, pomegranate, banana, custard apple.

Q) Which Navy has officially inducted the INS Sindhuvir that was gifted by India?

A. Vietnamese Navy B. Myanmar Navy C. Bangladesh Navy D. Philippine Navy

ANS - B

The Myanmar Navy has officially inducted the submarine INS Sindhuvir, which was handed over by the Indian Navy to the country in October 2020. This is the first submarine in the Myanmar Navy’s

103 TELEGRAM LINK: https://t.me/opdemy WEBSITE: www.opdemy.com

arsenal. The submarine has been refurbished for the Myanmar Navy by state-run defence shipbuilder, Hindustan Shipyard Limited (HSL) in Visakhapatnam.

Q) Bhabha Atomic Research Centre (BARC), Mumbai has developed the first indigenous Ruthenium 106 Plaque for treatment of Ocular Tumours which is built in which part of the body?

A. Blood B. Eye C. Lung D. Skin

ANS - B

Ocular tumors are tumors inside the eye. They are collections of cells that grow and multiply abnormally and form masses.

Q) What is the name of the stadium in Qatar that is built by an Indian company, Larsen & Toubro, and was recently visited by the External Minister S. Jaishankar during his trip to Qatar?

A. Lusail Stadium B. Al Bayt Stadium C. Al Janoub Stadium D. Ahmed bin Ali Stadium

ANS - D

Q) Which of these statements are correct?

1) Bihar has won the Digital India Award 2020 in the "Pandemic Category". 2) The award was given by the Central Government for the healthcare initiatives provided by the Bihar Government.

A. 1 only B. 2 only C. Both 1 and 2 D. Neither 1 nor 2

ANS - B

The award was given by the Central Government for the initiative of cash transfer to the beneficiaries through Bihar Parvasi Sahayta Mobile App.

104 TELEGRAM LINK: https://t.me/opdemy WEBSITE: www.opdemy.com

Q) Prime Minister Narendra Modi dedicated to the nation the ambitious New Bhaupur-New Khurja section of Eastern Dedicated Freight Corridor (EDFC) in:

A. Uttar Pradesh B. Bihar C. Punjab D. Rajasthan

ANS - A

Prime Minister Narendra Modi dedicated to the nation the ambitious New Bhaupur-New Khurja section of Eastern Dedicated Freight Corridor (EDFC) in Uttar Pradesh. The 351-kilometre New Bhaupur- New Khurja section of EDFC is built at a cost of Rs. 5,750 crore and most of its part passes through Uttar Pradesh. The section will decongest the existing Kanpur- Delhi mainline and will enable Indian Railways to run faster trains.

*************************************************************************************************************

Be True to Yourself, Have Patience, Your efforts will bear fruit!

All The Best !

105 TELEGRAM LINK: https://t.me/opdemy WEBSITE: www.opdemy.com